4 5821337585678549749 PDF

You might also like

Download as pdf or txt
Download as pdf or txt
You are on page 1of 75

‫مسؤول اللجنة‬ ‫مسؤولة اللجنة‬

‫نصر علي القادري‬ ‫آية محمود زيدان‬


‫اﻹﺧﺘﺒﺎر اﻟﻨﻬﺎﺋﻲ‬
‫ﻟﻌﺎﻣﻴﻦ ﺳﺎﺑﻘﻴﻦ‬
Pathology Exams ‫ ﻃﺐ ﺑﺸﺮي‬32 ‫اﻟﻠﺠﻨﺔ اﻟﻌﻠﻤﻴﺔ ﺑﺪﻓﻌﺔ ﻧﺒﺾ اﻟﺤﻴﺎة‬
‫بسم هللا الرحمن الرحيم‬
UNIVERSITY OF SANA’A- FACULTY OF MEDICINE
DEPARTMENT OF PATHOLOGY
FINAL EXAMINATION OF PATHOLOGY – THIRD YEAR MEDICAL STUDENTS 2015-2016
(November 2016) (time allowed: 3 hours)- total marks (120)
Part-1: choice (encircle) the most correct answer (only one) – from 1-49 (49 marks)
1. The important cause of chronic bronchitis is:
a. Decreased elastase activity
b. Alpha 1 antitrypsin deficiency
c. Smoking
d. Increased elastase activity
2. Adenocarcinoma of the lung usually has
a. Peripheral location
b. Central location
c. Both the above locations
d. None of above
3. Central emphysema affects:
a. Whole acinus
b. Distal acinus
c. Central and distal
d. Proximal acinus
4. Family history of allergy is seen in patients with:
a. None atopic asthma
b. Atopic asthma
c. Bronchial asthma
d. None bronchial asthma
5. Suppurative inflammation is a characteristic feature of:
a. Acute bronchitis
b. Bronchial asthma
c. Bronchopneumonia
d. Labor pneumonia
6. Ghon focus of granulomatous inflammation is a feature of:
a. Sarcoidosis
b. Primary T.B.
c. Secondry T.B.
d. Progressive T.B.
7. Kawasaki disease:
a. Affects infants

‫ﻣﺮﻛﺰ اﻟﺨﻠﻴﺞ اﻟﻌﺮﺑﻲ‬ 1


Pathology Exams ‫ ﻃﺐ ﺑﺸﺮي‬32 ‫اﻟﻠﺠﻨﺔ اﻟﻌﻠﻤﻴﺔ ﺑﺪﻓﻌﺔ ﻧﺒﺾ اﻟﺤﻴﺎة‬
b. Affects children
c. Affects infants and children
d. Affects old age
8. Aneurysm may result in all the following except one:
a. Thrombosis
b. Hemorrhage
c. Death
d. Atherosclerosis
9. Liner fatty streaks is the characteristic feature of
a. Early phase of AS.
b. Late phase of AS
c. Complicated phase of AS.
d. None of the above.
10. MI is characterized by localized:
a. Caseous necrosis
b. Coagulative necrosis
c. Liquefative necrosis
d. Fibrinoid necrosis
11. Chronic venous congestion is a feature of:
a. Left side HF
b. Right side HF
c. Renal failure
d. Angina pectoris
12. Angina pectoris is caused by:
a. Acute ischemia
b. Chronic ischemia
c. Decreased blood supply
d. Increased blood supply
13. Cardiomyopathy refers to:
a. Heart muscle disease
b. Cardiac inflammation
c. Valvular calcification
d. Valvular stenosis
14. A 30-year-old male noticed a progressive cough for one month. On physical examination, a
few small lymph nides were palpable in the axillae, and the tip of the spleen was palpable. A
CBC showed: Hb 10.2, Hct 31.1, MCV 90, WBC count 67000 and platelet count 36000. If blasts
with Auer rods are seen in the peripheral blood smear, then the most likely diagnosis is:
a. Acute myeloblastic leukemia
b. Multiple myeloma
c. Chronic lymphocytic leukemia
d. Acute lymphoblastic leukemia

‫ﻣﺮﻛﺰ اﻟﺨﻠﻴﺞ اﻟﻌﺮﺑﻲ‬ 2


Pathology Exams ‫ ﻃﺐ ﺑﺸﺮي‬32 ‫اﻟﻠﺠﻨﺔ اﻟﻌﻠﻤﻴﺔ ﺑﺪﻓﻌﺔ ﻧﺒﺾ اﻟﺤﻴﺎة‬
15. Regarding hodgkin’s lymphoma, which of the following is true:
a. Neoplastic disorder of histiocytes
b. T-cell type have worse prognosis
c. Lymphocyte depletion is the most common type.
d. Lymphocyte predominance type have excellent prognosis.
16. Painless lymphadenopathies in 5-year old anemic child cinsistent with:
a. Burkitt’s lymphoma
b. Acute myeloblastic leukemia
c. Myelodysplastic disorder
d. Acute lymphoblastic leukemia
17. The appearance of a ”target cell” on examination of the peripheral blood smear is most
consistent with origin from a patient who has:
a. Adenocarcinoma of the colon
b. A diagnosis of beta-thalassemia
c. Septicemia with E. coli
d. Hereditary spherocytosis
18. Anemia-bleeding disorders and infections are seen in the following diseases except:
a. Aplastic anemia
b. Leukemias
c. Iron deficiency anemia
d. Hypersplenism
19. A 35-year old anemic female presents with pneumonia and epistaxis with several purpuras.
Physical examination reveals no organomegaly. A bone marrow aspiration demonstrates dry
tap and bone marrow biopsy shows yellow marrow with band of fibrous tissue and sheets of
lymphocytes. Which set of peripheral blood estimation findings is most likely present:
a. CBC with Hb 8 gm/dl, WBC total 2000 per microliter, platelet count 3000 per
microliter.
b. CBC with Hb 20 gm/dl, Hct 61%, and MCV 92.
c. Total lymphocytes count of 200 per microliter, WBC 12000 per microliter.
d. CBC with Hb 10 gm/dl, Hct 30%, MCV85, platelet count 300000 per microliter.
20. The following are clinical consequences of liver disease except:
a. Jaundice
b. Hypercalcemia
c. Hypoalbuminemia
d. Palmer erythema
21. A 41-year old male with a history of chronic hepatitis has massive hematemesis with
prolonged prothrombin time. This is most typical for:
a. Portal hypertension
b. Reflux esophagitis
c. Barrett’s esophagus
d. Esophageal carcinoma

‫ﻣﺮﻛﺰ اﻟﺨﻠﻴﺞ اﻟﻌﺮﺑﻲ‬ 3


Pathology Exams ‫ ﻃﺐ ﺑﺸﺮي‬32 ‫اﻟﻠﺠﻨﺔ اﻟﻌﻠﻤﻴﺔ ﺑﺪﻓﻌﺔ ﻧﺒﺾ اﻟﺤﻴﺎة‬
22. Regarding seromarkers of HBV, which of the following is incorrect:
a. HbsAg represent glycoprotein and detected in the serum
b. HBC Ag represent core nucleocapsid and remains in the infected hepatocyte.
c. HBV-X protein represent precore and core region antigen and indicate progression to
chronic hepatitis.
d. HBV-X antigen represent the transformation to malignancy.
23. Which of the following drug overdose, produce massive hepatic necrosis:
a. Aspirin
b. Tetracycline
c. Retinol
d. Paracetamol
24. A 55-year old male who is hepatitis C seromarker positive has a firm, nodular liver. All the
following findings can occur as a complication of this condition except:
a. Hepatocellular carcinoma
b. Coagulopathy
c. Ascites
d. Hepatic infarction
25. The laboratory biopsy from a 60-year-old female with abdominal mass, deep jaundice and
elevated serum CEA reveals poorly differentiated glandular structure which lined by atypical
pleomorphic cells and surrounded by desmoplastic stroma. The probable diagnosis for these
findings is:
a. Gastric adenoma
b. Pancreatic carcinoma
c. Hepatocellular carcinoma
d. Choristma
26. Acute a calculous cholecystitis occurs in the following conditions except:
a. The postoperative state after major non-biliary surgery.
b. Severe trauma
c. Malnutrition
d. Severe burns
27. A 60-year-old female had a cerebral infarction. Months later, a computed tomographic (CT)
scan shows a cystic area in her cerebral cortex the CT findings is a lesion that consequence of
resolution from:
a. Liquefactive necrosis
b. Apoptosis
c. Caseous necrosis
d. Atrophy
28. Which of the following infectious agents is the most likely to produce focal necrotizing
encephalitis:
a. Herpes simplex virus
b. Toxoplasma gondii
c. Cytomegalovirus
‫ﻣﺮﻛﺰ اﻟﺨﻠﻴﺞ اﻟﻌﺮﺑﻲ‬ 4
Pathology Exams ‫ ﻃﺐ ﺑﺸﺮي‬32 ‫اﻟﻠﺠﻨﺔ اﻟﻌﻠﻤﻴﺔ ﺑﺪﻓﻌﺔ ﻧﺒﺾ اﻟﺤﻴﺎة‬
d. Bacteroides
29. Examination of the CSF from labor puncture shows 2 RBC’s, 34000 WBC, glucose of 20mg/dl,
and protein of 105 mg/dl. Which of the following additional tests would be the most helpful:
a. Cryptococcal antigen
b. Acid fast stain
c. Gram stain
d. India ink
30. A 65-year-old male has been healthy all his life until a sudden seizure. Neurologic exam
reveals no focal abnormalities. A CT scan reveals a poorly demarcated large mass with
central necrosis in the right frontal lobe. The most likely diagnosis is:
a. Glioblastoma multiforme
b. Medulloblastoma
c. Low grade astrocytoma
d. meningioma
31. the most common cause of dementia on the elderly population is:
a. Parkinson disease
b. Alzheimer disease
c. Demyelinating disease
d. Huntington disease
32. A 13-year-old boy with pain in his left thigh is found to have a neoplasm of the femur that
radiologically is diaphyseal in location and in biopsy shows numerous small round blue cells.
The probable diagnosis is:
a. Chondrosarcoma
b. Metastatic adenocarcinoma
c. Ewing’s sarcoma
d. Neuroblastoma
33. The most common causative bacteria for acute osteomyelitis is:
a. E. coli
b. Staphylococcus aureus
c. Proteus
d. Salmonella
34. An 80-year-old woman falls out of bed and breaks her hip. Radiographs show not only a
fracture of the left femoral head, but also a compressed fracture of T10. She probably has:
a. Osteitis deformans
b. Osteopetrosis
c. Ankylosing spondelitis
d. Osteoporosis
35. A 39-year-old male has a slightly raised pigmented lesion on the chest which has recently
become darker and enlarged, with irregular borders present. This should suggest that it is a:
a. Malignant melanoma
b. Lentigo nevus
c. Intradermal nevus
‫ﻣﺮﻛﺰ اﻟﺨﻠﻴﺞ اﻟﻌﺮﺑﻲ‬ 5
Pathology Exams ‫ ﻃﺐ ﺑﺸﺮي‬32 ‫اﻟﻠﺠﻨﺔ اﻟﻌﻠﻤﻴﺔ ﺑﺪﻓﻌﺔ ﻧﺒﺾ اﻟﺤﻴﺎة‬
d. Basal cell carcinoma
36. The worldwide increase in skin cancers has resulred primarily from increased exposure to:
a. Ultraviolet B (UVB) radiation
b. Gamma radiation
c. Nitrous oxides (NOx)
d. Chlorofluorocarbons (CFC’s)
37. Hemorrhagic and slightly enlarged adrenals wre found at autopsy in a teenager who died
only hours after presentation to the emergency room with fever and severe headache. This
appearance is most consistent with
a. Idiopathic addison’s disease
b. Metastatic carcinoma
c. Meningococcemia
d. Widespread tuberculosis
38. Diffuse non-toxic (simple) goiter used to be common (endemic) in mountain region because
of:
a. Vitamin D deficiency
b. Lack of dietary iodine
c. Lack of dietary calcium
d. Genetic lack of (thyroid binding globuin)
39. A 29-year-old primagravida has a placenta previa with extensive blood loss and shock during
delivery, she is most likely to have which of the following problems:
a. Cushing’s syndrome
b. Grave’s disease
c. Galactorrhea
d. Sheehan syndrome
40. Which of the following is high risk factor for adenocarcinoma of the esophagus:
a. Tobacco
b. Very hot beverages
c. Chronic alcoholism
d. Barrett’s esophagus
41. Morphological sighs of glomerulonephritis is:
a. Tubular necrosis
b. Interstitial fibrosis
c. Increase glomerular cellularity
d. Rupture ascending tubules
42. Morphology of pyelonephritis:
a. Interstitial inflammation
b. Increased glomerular mesangium
c. Cortical fibrosis
d. Hyperplasia of transitional cells
43. Acute nephritic syndrome
a. Due to aflatoxins
‫ﻣﺮﻛﺰ اﻟﺨﻠﻴﺞ اﻟﻌﺮﺑﻲ‬ 6
Pathology Exams ‫ ﻃﺐ ﺑﺸﺮي‬32 ‫اﻟﻠﺠﻨﺔ اﻟﻌﻠﻤﻴﺔ ﺑﺪﻓﻌﺔ ﻧﺒﺾ اﻟﺤﻴﺎة‬
b. End-stage kidney
c. Causes polyuria
d. Leads to anuria
44. Male germ cell tumors:
a. Arise from Sertoli cells
b. Arise from Leydig cells
c. Arise from seminiferous tubules
d. Arise from immature cells
45. Chronic renal failure:
a. Developed in short time
b. Renal function remain normal
c. The patients have normal urine
d. Lead to end-stage kidney
46. Renal calculi are:
a. Uric acid 70%
b. Calcium contain stone 75%
c. Cysteine 50%
d. Mixed material (phosphate and magnesium)
47. Renal cell carcinoma:
a. Invades renal veins
b. Metastasize to other kidney
c. Small round cell neoplasm
d. Never give metastasis
48. Prostatic hyperplasia:
a. An inflammatory process
b. Leads to carcinoma
c. Disease of old age male
d. A tumor of male genital tract.
49. Prostatic carcinoma:
a. Dose not give metastasis
b. Arise from peripheral zone
c. Common in males before 50 years
d. Arise from central part of gland.

(50-54) match the letter with appropriate number (5 marks)


A. Ulcerative colitis B. Crohn disease C. both D. none of them
( ) 50. Associated with right sided heart failure
( ) 51. Confined to large intestine
( ) 52. Any site of the gastrointestinal tract can be affected

‫ﻣﺮﻛﺰ اﻟﺨﻠﻴﺞ اﻟﻌﺮﺑﻲ‬ 7


Pathology Exams ‫ ﻃﺐ ﺑﺸﺮي‬32 ‫اﻟﻠﺠﻨﺔ اﻟﻌﻠﻤﻴﺔ ﺑﺪﻓﻌﺔ ﻧﺒﺾ اﻟﺤﻴﺎة‬
( ) 53. Granulomatous inflammation is 60% of the cases
( ) 54. Inflammatory bowel diseases
(55-60) false and true (6 marks)
( ) 55. One of the benign ovary tumor is struma ovarli.
( ) 56. The malignant surface epithelial ovary tumors show an infiltrative growth.
( ) 57. Hemorrhage cyst is main pathological feature of pvary follicular cyst.
( ) 58. Chorionic villi, atypical trophoblasts proliferation and myometrium invasion are feature
of choriocarcinoma.
( ) 59. Partial vesicular mole associated with metastasis.
( ) 60. Fallopian tuve is very commonest site of ectopic pregnancy.

Part II: Essay questions- answer all the following questions (60 marks- 5 marks for each branch)
Q1. A. Write the major manifestations of rheumatic fever?
B. Write short account on squamous cell carcinoma of the lung?
Q2. A. Enumerate the causes of pancytopenia.
B. Write short notes on the morphology of secondary biliary cirrhosis.
Q3. A. Write short notes on the classification of bone neoplasms.
B. write short account about brain abscess (causative organism, morphology and
complications)
Q4. A. Enumerate the thyroid neoplasms benign and malignant.
B. Write short notes on gastric carcinoma
Q5. A. Write short notes about nephrotic syndrome.
B. Classify tumors of testis and write short notes about seminoma.
Q6. A Define the following: - Adenomyosis – Intracanalicular fibroadenoma – CIN-1
- Complex endometrium hyperplasia.
B. Writhe short notes about microscopic picture of the following:
- Endometrium polyp.
- Breast fibroadenoma.
- Simple endometrium hyperplasia.

‫ﻣﺮﻛﺰ اﻟﺨﻠﻴﺞ اﻟﻌﺮﺑﻲ‬ 8


Pathology Exams ‫ ﻃﺐ ﺑﺸﺮي‬32 ‫اﻟﻠﺠﻨﺔ اﻟﻌﻠﻤﻴﺔ ﺑﺪﻓﻌﺔ ﻧﺒﺾ اﻟﺤﻴﺎة‬

final Exam of Pathology


. Third Y.t?ar, �P1t
Tfme-affowed: 1,Sh
Choice the correct answer ( only one ):

1� Endometrialbiopsy should be taken:


A.. �yrif'l�\�h�rprqlifetativepl'lase forthe·evaluation of vaginal bleeding
B- a�th� we111�pstrual period for assessment of ovulation
C- on the 14th day for assessment of ovulation
0- during the menstrual period for all diagnostic purposes

2- Peptic ulcer affects:


A-duodenum 8-esophagus C- colon D- rectum

3- All of the following are true regardi.ng Lupus nephritis, except:


A- kidney inflammatory disease B- autoimmune disease
C- manifestation of systemic lupus erythematosus D- malignant

4.. All of the following are false regarding intrinsic bronchial asthma, except:
A- it is assodated with a family history of allergy
B- it is mediated by a type one hypersensitivity reaction
C- no evidence of atopy D- starts in childhood

5.. All of the following are pathological features of bronchial asthma, except:
A- thickening of the basement membrane B- bronchial squamous metaplasia
C- hypertrophy of the bronchial muscle D- edema

6� All have no perimalignant potential eJ<cept:


A- lichen simplex atrophicus B- lichen simplex
c� herpes simplex. D- condylomatum latum

7- Sheehan's Syndrome is:


A- adrenal hemorrhage 8- ischemic necrosis of the myocardium
C- gangrene of lower extremities
D- postpartum ischemic necrosis of the anterior pituitary gland

Sa All of the following are true regarding lo�ar pneumonia, except:


A- affects young males B- fibrino- suppurative exudate
C- consolidation of entire lobe D- only suppurative exudae

9a Painless lymphadenopathies in five years old anemic child consistent with :


A- acute lymphoblastic leukemia B� acute myeloblastic leukemia
C- burkitt lymphoma D- localized plasmacytoma

‫ﻣﺮﻛﺰ اﻟﺨﻠﻴﺞ اﻟﻌﺮﺑﻲ‬ 9


Pathology Exams ‫ ﻃﺐ ﺑﺸﺮي‬32 ‫اﻟﻠﺠﻨﺔ اﻟﻌﻠﻤﻴﺔ ﺑﺪﻓﻌﺔ ﻧﺒﺾ اﻟﺤﻴﺎة‬

10- Prolong partial thromboplastin time, hemoarthrosis and absence of petechiae


are characteristic features for:
A- chronic liver disease B- hemophilia-A
C-desiminated intravascular coagulation D- hemoph.ilia-B

11- Which of the following is primitive ineuroectodremal tumor ( PNET):


A- astroCytoma B- medulloblastoma
C- ependymoma D- glioblastoma

12; vrnJas tn�ningitis characterized by :


A� n?4tr9pbJlic exudate 8- cyst in t�e cerebrum
C- granulomas D- perivascular lymphocytes

l�·Vagina is characterized by:


A- maHg11�nt lesions are more common than benign lesions
8- common site for infection C- Poor in bacteria flora
D- secondary tumors are more common than primary

14· Regarding Hodgkin lymphoma whic:h of the following. is true:


A- neo,plastic disorders of histiocytes
B- T-cell type have worse prognosis
C- lymphocyte predominance type have ·excellent prognosis
Du· lymphocyte depletion is the most common type

15.. Regarding the primary malignancies of liver, which of the following is not
carcinogenic:
A.._ HAV B- inorganic arsenicals C- aflatoxin D-HCV

16- Neisseria meningitis characterized b'V:


A- granulomas B- cyst in the cerebrum
C- neutrophils D- .perivascular lymphocytes

li- Non caseating granuioma Is a featun� of:


A- gastric ulcer 8- tuberculosis C- ulcerative colitis D- Crohn's disease

1s.. All of the following are ovarian neoplasms except:


A- serous cystadenoma B- follicular cyst
C- sertoli cell tumor D- granulosa celf tumor

19- All of the following are types of renal! cell carcinoma, ,except:
A- clear cell type B- follicular cell type
C- papillary type D-chromophobe type

20.. A 45 years old male has skin infiltration by neoplastic T lymphocytes� His
condition is known as:
A- burkitts lymphoma B- mycosis fungoides
C-Hedgkin lymphoma D- acute lymphocytic leukemia

‫ﻣﺮﻛﺰ اﻟﺨﻠﻴﺞ اﻟﻌﺮﺑﻲ‬ 10


Pathology Exams ‫ ﻃﺐ ﺑﺸﺮي‬32 ‫اﻟﻠﺠﻨﺔ اﻟﻌﻠﻤﻴﺔ ﺑﺪﻓﻌﺔ ﻧﺒﺾ اﻟﺤﻴﺎة‬

21� CIN is characterized by:


A- high grade lesions almost always prograss to invasive cancer
. 8- high grade lesions may c:ompletely regress
C- all arise in the transformation zone D- ·sexually transmitted disease

22� Primary p1.1lmonary TB mostly affects:


A- adults 8- children C- old age D- elderly

23u The following are primary hepatic neoplasms, except:


A- hepatoblastoma B-mesotheliotna C-hepatoma D- adenoma

24- ovarian tumors:


A- present e�rly 8- present late
C- ascetic cytology is diagnostic D- CT scan provide specific diagnosis

25m The following are true about the causes of death in cirrhotic patients, except:
A- cholangiocarcinoma 8- liver faHure
C- massive hematamesis · D- hepatic coma

z5 .. Myocardial infarction characterized by:


A- localized ischemic necrosis B- Diffuse ischemic necrosis
c� abscess D- fibrosis

'2.7- Lymphoma:
A- malignant 8- benign C- commonly solitary D- rare

28- Commonest cause o'f pyelonephritis:


A- obstruction 8- blood born infection
C- tubular necrosis D- interstitial cystitis

29.. Acute bacterial endocarditis:


A- affects damaged valves B- affects normal valves
C- affects joints 0- affects skin

30- Papillary thyroid carcinoma is histologkaily diagnosed by:


A- metastasis 8- capsular invasion c- vascular invasion D- nuclear features

31� Sertoli cell tumor of testiis is:


A- a non germ cell tumor 8- malignant
c� germ cell tumor D- laydig cell tumor

32 .. Advanced gastric carcinoma is commonly spread by:


A� local extension B- blood C- lymphatics D- sputum

33- All of the following are thyroid neoplasms, except:


A- papillary carcinoma Bw follicular carcinoma
Cw hepatoma D- thyroid adenoma,

‫ﻣﺮﻛﺰ اﻟﺨﻠﻴﺞ اﻟﻌﺮﺑﻲ‬ 11


Pathology Exams ‫ ﻃﺐ ﺑﺸﺮي‬32 ‫اﻟﻠﺠﻨﺔ اﻟﻌﻠﻤﻴﺔ ﺑﺪﻓﻌﺔ ﻧﺒﺾ اﻟﺤﻴﺎة‬

34.. All of the following complications are false regarding vasculitis, except:
A- aneurysm 8- atherosclerosis C- ischemia D- thrombus

35- Prosfatlc hyperplasia-:


A- chronic B- malignant C- neoplastic D- non neoplastic

36- Commonly site of the duodenal ulcer is:


A- posterior wall B- anterior wall C- cardia D- lesser curvature

3·1�:'Psarnrnomabodies seen··in:
A- astrocytoma B� oligodendroglioma C- ganglioma D- meningioma ·

38- Most cases of colorectal cancer ocicur in the:


A- colon 8- rectum C- colorectum D- cecum

39- Dissecting aneurysm is associated with:


A- smoking B- medionecrosis C- infection D- hypotension

40.. Hepatoma:
A- germ cell neoplasm B- sertoli cell neoplasm
C- n'on neoplastic D- malignant

41- Alf of the following are true regarding emphysema, except:


A- permanent dilation B- permanent contraction
C- form of COPD D- affects air ways

42- All of the following are true regardiing large cell carcinoma of the lung, except:
A- small tumor mass B- central location
C- peripheral location D- large tumor mass

43m Of the following factors one is not predisposing fa�tor for atiherosderosis:
A- air pollution B- diabetes
. C- hyperlipldemia D- hypertension

44- All of the following are pathological features of chronic bronchitis, except:
A- bronchial muscular hypertrophy B- hyperplasia of mucous glands
C- hypersecretion of mucus D- bronchial squamous metaplasia

45- All of the following are true regarding., embryonal carcinoma of the testis
, except:
A- more aggressive than seminoma B· usually unaggressive
C- uncommon D- affects male

46� Goiter is:


A· adenoma of thyroid gland B- metastatic malignant tumor
�- papillary thyroid carcinoma D- non neoplastic lesion of thyroid gland

‫ﻣﺮﻛﺰ اﻟﺨﻠﻴﺞ اﻟﻌﺮﺑﻲ‬ 12


Pathology Exams ‫ ﻃﺐ ﺑﺸﺮي‬32 ‫اﻟﻠﺠﻨﺔ اﻟﻌﻠﻤﻴﺔ ﺑﺪﻓﻌﺔ ﻧﺒﺾ اﻟﺤﻴﺎة‬

41- Nephrotic syndrome characterized by:


a-hypblipidemia b- hematuriai c� massive proteinuria dM aJI the above

48- GiantceH arteritis affects':


A- giant cells 8- adults C- old age D- infants

49a All of the following are true regarding nodular goiter, except:
A- hyperplastic B- may be multiples
C-cornmonly single nodule D- infiJtrative

5Prt\r,gina.pectoris is.one;! �y,pe of:


A�MI B- SCD C- RF D-IHD

51- All of the following are true regarding medial calcified sclerosis
, except:
A- no clinical manifestations B- calcified changes in the media
C- affects old age D- affects young age

52- Early phase of atherosclerosis characterized by:


A- linear fatty streaks 8- formation of intimal nodule
C- formation of medial nodule D- hyafine changes

53w All of the foHowing are f'.alse regarding Kawasaki disease ,except:
A- seen in infants B- seen in young women
C- associated with smoking D- severe pain

54.. All of the following are major fea1tures of rheumatic fever, except:
A- pericarditis 8- polyatreritis
C- subcutaneous nodules D- chorea

55- Hyperplastic type of cardiomyopathy results from.:


A- hypertension B- myocardial infarction
C- obstruction of the outflow from the left ventricle D- IHD

56- Hemopericardium means accumulation of:


A- blood in the pericardium 8- fluid in the pericardium
C- serous fluid in the pericardium D- exudate in the pf eura

57- All of the following are true regarding aneurysm ,except:


A- dilation of blood vessel wall 8- dilation of myocardial wall
C- results in rupture De usually affects young age

58m Nephritic syndrome characterized by;


A- heavy proteinuria 8- polyuria
C- lipiduria D- azotemia

‫ﻣﺮﻛﺰ اﻟﺨﻠﻴﺞ اﻟﻌﺮﺑﻲ‬ 13


Pathology Exams ‫ ﻃﺐ ﺑﺸﺮي‬32 ‫اﻟﻠﺠﻨﺔ اﻟﻌﻠﻤﻴﺔ ﺑﺪﻓﻌﺔ ﻧﺒﺾ اﻟﺤﻴﺎة‬

59- Burkitt's lymphoma:


A- high grade B- intermediate grade
C- T-cell origin lymphoma D- small cleaved cells

60�Cofbovinum results from:


A- disse<::ting aneurysm 8- syphilitic aneurysm
C- atherosclerotic aneurysm D- false aneurysm

61- AUPf.�h� _fglle>wing �r� trLle regarding endqcervicaJ pofyp,'!�-c::,rJ7


Ai m'�ss'cif'ffiilfignant tumor benign lesion
B- mass of
c- no malignant potentia l · D- uncertain cause
ij2� Endometrial hyperplasia results; from hormonal disturbance:
A- increase in progesterone 8- decrease in progesteone
C- increase in estrogen D- decrease in estrogen

63- Transformation zone is characterized· by all of the following except:


A- it is low down before puberty B- it is high up in menopausal women
C- it is the site of origin of cervical precancerous lesions.
D- .Pap smear is inadequate if I.doesn't sample this zone

64- Distal emphysema:


A- affects lower fobes 8- associated with smoking
C- occurs in alveolar ducts and alveoli D- affects upper lobes

65- AU of the following are features of progressive pulmonary tuberculosis,


,except:
A- hemoptysis B- acute miliary tuberculosis
C- Tuberculous ulcer of intestine D- secondary pulmonary tuberculosis

66D - Ghon complex is a feature of:


A- primary pulmonary TB a:�chronic bronchitis C- lung carcinoma D-granuloma

67- Leoomyoma is:


A- commonly multiple B- commonly single
C- its growth is stimulated by location D- encapsulated

68- Small cell lung carcinoma:


A- large tumor cells B- aggressive C- unaggressive D- mesenchymal

69- Common malignant tumor in aduft kidney:


Aw oncocytoma B- urothelial carcinoma
C- nephrobf astoma C- clear cell carcinoma

70- All of the following are true regarding Pheochromocytoma, ,except:


.. A- most often benign
C- tumor of adrenal medulla
B- some are malignant
D- tumor of kidney

‫ﻣﺮﻛﺰ اﻟﺨﻠﻴﺞ اﻟﻌﺮﺑﻲ‬ 14


Pathology Exams ‫ ﻃﺐ ﺑﺸﺮي‬32 ‫اﻟﻠﺠﻨﺔ اﻟﻌﻠﻤﻴﺔ ﺑﺪﻓﻌﺔ ﻧﺒﺾ اﻟﺤﻴﺎة‬

71- Golmerulonephritis is a disease 1:>f


A- cortex 8- medulla C- glomerUli D- tubules

· 72�,A11:romeg�Jy is:
A- benign tumor of the lung B- aduJt onset excess growth hormone
C- metastasis of bronchogenic carc,inoma o,. ma}igrlanttumor of the skin
73a �napla�tic thyroid carcinoma ( undifferentiate�ff�rrfinon,a ):
A- ���
. .•
�·-· ��r,yp��.
r.�r. ��nosis
. ���gg?p�?.fr.R,f'�
c>tne mOsf coH'imon ·thyroid cancer ck erlcapsi:ilat��>turnor
74� .Dill of the following are true regarding risk factOrs for renal celhc�rciqpma
, except:
A- smoking B- obesity C- hypertension D- cystic bladder changes

75m Nephritic syndrome characterized! by;


A- heavy proteinuria 8- polyuria
C- lipiduria D- azotemia

76.. All of the following are types of n<>n infectious v,sculitis, except:
A- Kawasaki disease B- thromboangitis obl.iterans
C- syphilitic arteritis D- polyarteritis nodosa

77- Neuroblastoma is common seen in:


A- adults 8- old age C- children D- elderly

· 78- Rheumatic fever:


A- inflammatory disease 8- non inflammatory disease
B- manifestation of Ml D- autoimmune disease

‫ﻣﺮﻛﺰ اﻟﺨﻠﻴﺞ اﻟﻌﺮﺑﻲ‬ 15


‫ﻧﻣﺎذج ﻣﻘﺳﻣﺔ‬
‫ﺣﺳب‬
‫اﻟﻣواﺿﯾﻊ‬
Pathology Exams ‫ ﻃﺐ ﺑﺸﺮي‬32 ‫اﻟﻠﺠﻨﺔ اﻟﻌﻠﻤﻴﺔ ﺑﺪﻓﻌﺔ ﻧﺒﺾ اﻟﺤﻴﺎة‬

Cardio pathology
Give short account on:
1. Aneurysm (definition-complications) and talk about the most common type
2. Arteriosclerosis
3. Complications and risk factors of atherosclerosis
4. Varicose
5. Stable angina
6. Trans-mural myocardial infraction
7. Myocardial infraction(def-path-microscopic &gross picture –complication)
8. What are the main differences between
- Kawasaki and Takayasu disease
- Thromboangitis obliterans and poly arteritis nodosa
- Trans-mural MI and Sub-endocardial infarction
9. Classification of aneurysm according to (pathogenesis - Shape - The
composition of the aneurysm )
10.Give the definition and cause of the following :
a) Marantic endocarditis
b) Sudden cardiac death
11.Write the major manifestation of rheumatic fever
12.Left side heart failure ( def.- causes .morphology)

MCQs :

1.Cardiactoamponade mean: (not clear=sorry)


a) Hemo-pericardium.
b) Hydro-pericardium.
c) Cardiomyopathy
2.Atherosclerosis is slowly progressive disease marked by:
a) Elevated intimal fibro fatty plaque
b) Proliferation of intima ,media and fibrosis
c) Medial sclerosis of femoral artery
d) Non-of the above
3.Aneurysm is:
a) dilation of lower limb vein
b) localized abnormal dilation usually artery
c) rupture of blood vessels
d) all of the above

‫ﻣﺮﻛﺰ اﻟﺨﻠﻴﺞ اﻟﻌﺮﺑﻲ‬ 16


Pathology Exams ‫ ﻃﺐ ﺑﺸﺮي‬32 ‫اﻟﻠﺠﻨﺔ اﻟﻌﻠﻤﻴﺔ ﺑﺪﻓﻌﺔ ﻧﺒﺾ اﻟﺤﻴﺎة‬

4.Myocardial infarction is mostly caused by :


a) atheromatous plaque
b) cor pulmonale
c) pulmonary embolism
d) all of the above
5.Atherosclerosis is characterized by formation of atheroma within :
a) media
b) intima
c) adventitia
6.Kawasaki disease
a) affect infant
b) affect children
c) affect infant and children
d) affect old age
7.Aneurysm may result in all the following except one :
a) thrombosis
b) hemorrhage
c) death
d) atherosclerosis
8.Aschoff bodies are present in :
a) rheumatic pericarditis
b) myocardial infarction
c) acute bacterial endocarditis
d) all of the above
9.Chronic venous congestion is feature of :
a) left side H.F
b) right side H.F
c) renal failure
d) angina pectoris
10.Angina pectoris is caused by :
a) acute ischemia
b) chronic ischemia
c) decrease blood supply
d) increase blood supply
11.Cardiomyopathy refers :
a) heart muscle disease
b) cardiac inflammation

‫ﻣﺮﻛﺰ اﻟﺨﻠﻴﺞ اﻟﻌﺮﺑﻲ‬ 17


Pathology Exams ‫ ﻃﺐ ﺑﺸﺮي‬32 ‫اﻟﻠﺠﻨﺔ اﻟﻌﻠﻤﻴﺔ ﺑﺪﻓﻌﺔ ﻧﺒﺾ اﻟﺤﻴﺎة‬

c) valvular calcification
d) valvular stenosis
12.Non infection vasculitis >> syphilitic arterial
13.Kawasaki all false except >>effect infant
14.gaint cell arteritis >>effect old
15. aneurysm all true except >>young
16.dissection aneurysm>>medionecrosis
17.cor bovius >>syphilitic arteritis
18.medical calcific sclerosis all except >>young age
19.early stage of atherosclerosis >>liner fatty streaks
20.rheumatic fever >>autoimmune inflammation
21. major feature of rheumatic fever except >>pericarditis
22.acute infection endocarditis >>normal valve
23.hemopericardia>> blood
24.angina >>IHD
25.MI>> localized ischemia
26.all predispose to atherosclerosis except>> air pollution
--------------------------------------------------------------------------------------------------------------------

Respiratory system
Essays :

1. Write short account of bronchial asthma.


2. Write short account in emphysema.
3. Mention the main type of lung carcinoma and write short account on squamous
cell type.
4. What the clinical feature of chronic bronchitis.
5. What the etiology of bronchiectasis.
6. Write short account on bronchiectasis.
7. Write short account on bronchial asthma (types, their differences and
pathogenesis of the most common type)
8. Classify pneumonia and write an essay about lobar pneumonia.

‫ﻣﺮﻛﺰ اﻟﺨﻠﻴﺞ اﻟﻌﺮﺑﻲ‬ 18


Pathology Exams ‫ ﻃﺐ ﺑﺸﺮي‬32 ‫اﻟﻠﺠﻨﺔ اﻟﻌﻠﻤﻴﺔ ﺑﺪﻓﻌﺔ ﻧﺒﺾ اﻟﺤﻴﺎة‬

MCQs :

1. Ghon’s complex occurs mainly in:


a) primary pulmonary TB
b) progressive pulmonary TB
c) secondary pulmonary TB
2. Acute inflamed bronchioles is seen in:
a) lobar pneumonia
b) bronchopneumonia
c) lobar and bronchopneumonia
3. miliary tuberculosis occurs as a result of:
a) blood vessel invasion
b) spread of infection to many organs
c) rupture of bronchus
4. extrinsic bronchial asthma is mediated by:
a) B-cell immune reaction
b) T-cell immune reaction
c) unknown mechanism
5. pulmonary infarction occurs as effect of:
a) moderate pulmonary emboli
b) large arterial emboli
c) moderate arterial emboli

6. bronchial muscle contraction is characteristic feature for:


a) chronic bronchitis
b) emphysemia
c) bronchial asthma
7. central location of lung carcinoma is characteristic feature for:
a) squamous cell carcinoma
b) small cell carcinoma
c) adenocarcinoma
8. lung adenocarcinoma is usually associated with:
a) smoking
b) scarring process
c) a-1 antitrypsin deficiency

‫ﻣﺮﻛﺰ اﻟﺨﻠﻴﺞ اﻟﻌﺮﺑﻲ‬ 19


Pathology Exams ‫ ﻃﺐ ﺑﺸﺮي‬32 ‫اﻟﻠﺠﻨﺔ اﻟﻌﻠﻤﻴﺔ ﺑﺪﻓﻌﺔ ﻧﺒﺾ اﻟﺤﻴﺎة‬

Hepato-biliary pathology:
Essays :

1. Enumerate hepatotropic viruses ?


2. What are the complications of acute and chronic cholecystitis ?
3. Enumerate the causes and complications of portal hypertension.
4. Mention the clinical syndrome produced by viral hepatitis.
5. Discuss the morphology of viral cirrhosis.
6. What are the risk factors of hepatocellular carcinoma ?
7. Describe the pathogenesis of acute pancreatitis .
8. Define and mention the clinical features of cholestasis .
9. Describe gross and microscopic change of liver cirrhosis.
10.Talk about primary carcinoma of liver.
11.What are complication of acute and chronic cholecystitis
12.Discuss morphology of viral cirrhosis
13.Describe pathogenesis of pancreatitis
14.Enumerate the cause and complication of portal hypertension
15.Write short notes on the morphology of secondary biliary cirrhosis

MCQs :

1. The following are clinical consequences of liver disease Except:


a) Hypocalcaemia
b) Jaundice
c) Hypoalbumenemia
d) Palmer erythema
2. 24-years0old medical student has a needle stick injury .A year later transaminase
are elevated and a liver biopsy shows collapse of liver lobules with bridging
necrosis ,and portal fibrosis marked infiltration by lymphocytes . The best
diagnosis is:
a) Acute B hepatitis
b) Primary biliary cirrhosis
c) Chronic active hepatitis
d) Acetaminophen overdose
3. Which statement regarding chronic hepatitis is correct :
a) Hepatitis A progress to chronic hepatitis in 5-10% of cases
b) Chronic persistent hepatitis is characterized by presence or (necrosis)

‫ﻣﺮﻛﺰ اﻟﺨﻠﻴﺞ اﻟﻌﺮﺑﻲ‬ 20


Pathology Exams ‫ ﻃﺐ ﺑﺸﺮي‬32 ‫اﻟﻠﺠﻨﺔ اﻟﻌﻠﻤﻴﺔ ﺑﺪﻓﻌﺔ ﻧﺒﺾ اﻟﺤﻴﺎة‬

c) Chronic active hepatitis is characterized by intact hepatic lobules


d) Auto Ab are detected by some patients with drug induced chronic hepatitis
e) Chronic persistent hepatitis is often progress o cirrhosis
4. Which of the following histology in the liver produced by constrictive pericarditis :
a) Macro-nodular cirrhosis
b) Portal lymphocytic infiltration
c) Bile duct proliferation
d) Sinusoidal dilatation
5. Which tumor mostly associated with use of oral contraceptives :
a) Bile duct adenoma
b) Hepatocellular carcinoma
c) Focal nodular hyperplasia
d) Hepatocellular adenoma
6. The following are clinical consequences of liver disease except:
a) hypercalcemia b) hypoalbuminemia c) jaundice d) Palmer erythema
7. Which of the following lesions is reversible hepatic injury:
a) fatty change b) liver cirrhosis c) massive hepatic necrosis d) infraction
8. All the following are complications of hepatic failure except:
a) coagulopathy b) hepato-renal syndrome
acute pancreatitis d) multiple organ failure
9. The following are clinical consequences of liver disease except:
a) jaundice b) hypoalbuminemia c) hypercalcemia d) Palmer erythema
10.The most common causes of jaundice include the following except:
a) hemolytic anemia b) hepatitis obstruction of flow of bile
e) autoimmune hepatitis
11.Regarding the primary malignancies of liver which of the following is not
carcinogenic:
a) vinyl chloride b) alpha-toxin c) HAV d) inorganic arsenide(not-clear)
12.The following are inborn errors of metabolism and pediatric liver disease except:
a) hemochromatosis b) Wilson’s disease
c) hepatoblastoma d) alpha-1 antitrypsin deficiency
13.Gall bladder stone :
a) due to hemolytic b) common cause of acute cholecystitis
d) mostly of cholesterol type d) common in adult female

‫ﻣﺮﻛﺰ اﻟﺨﻠﻴﺞ اﻟﻌﺮﺑﻲ‬ 21


Pathology Exams ‫ ﻃﺐ ﺑﺸﺮي‬32 ‫اﻟﻠﺠﻨﺔ اﻟﻌﻠﻤﻴﺔ ﺑﺪﻓﻌﺔ ﻧﺒﺾ اﻟﺤﻴﺎة‬

14.Hepatitis C virus :
A) common type in Yemen
b) more dangerous than B virus
c) it's vaccine is already available
d) it's define seromarker
15.Gall bladder anemia :
A) due to hemolytic anemia b) common cause of acute cholecystitis mostly
cholesterol type d) common in adult fatty
female
16.The following are ssRNA viruses except :
a)HAv b)Hbv c)Hbv d)Hcv
17.Elevated serum level of amylase and lipase are seen in:
a)acute calculous cholecystitis B)cystic fibrosis of pancreas
C)Chronic pancreatitis D)acute hemorrhagic necrotizing pancreatitis
18.The following are clinical consequences of liver disease except :
A) hypercalcemia B)jaundice
C)hypoalbuminemia D)Palmer erythematosus
19.What is of the following is true about ingestion of 20 g of acetaminophen:
a) Normal serum transaminase level
b) Spottynonzonal liver cell necrosis
c) The degree of injury is dose dependent
20.Which statement regarding chronic hepatitis is correct :
a) Hepatitis A progress to chronic hepatitis in 10 – 5%
b) Chronic persistence hepatitis is characterized by presence of placemnal necrosis
c) Chronic active hepatitis is characterized by intact hepatic lobules
d) Auto Ab are detected by some patient with drug induce chronic hepatitis
e) Chronic persistent hepatitis is often progress to cirrhosis#
21.Which of the following histology in the live produced by constrictive pericarditis :
a) Macro-nodular cirrhosis
b) Portal lymphocytic infiltration
c) Bile duct proliferation
d) Sinusoidal dilatation#
22.Which tumor most commonly associated with use of contraceptive :
a) Bile duct adenoma
b) Hepatocellular carcinoma
c) Focal follicular hyperplasia
d) Hepatocellular adenoma #

‫ﻣﺮﻛﺰ اﻟﺨﻠﻴﺞ اﻟﻌﺮﺑﻲ‬ 22


Pathology Exams ‫ ﻃﺐ ﺑﺸﺮي‬32 ‫اﻟﻠﺠﻨﺔ اﻟﻌﻠﻤﻴﺔ ﺑﺪﻓﻌﺔ ﻧﺒﺾ اﻟﺤﻴﺎة‬

23.The following are clinical consequences of liver diseases except :


A. Jaundice B. Hypercalcemia
C. Hypoalbuminemia D. palmer erythemia
24.A 41 –year –old male with a history of chronic hepatitis has massive hematemesis
with prolonged prothrombin time . This most typical for :
A. Portal hypertension B. Reflex esophagitis
C. Barrett's esophagus D. Esophageal carcinoma
25.Regarding seromarkers of HBV ,which of the following is incorrect:
a. HbsAg represent glycoprotein and detected in serum .
b. HBCAg represent core nucleocapside and remains in the infected hepatocyte .
c. HBV-X protein represent precore and core region antigen and indicate
progression to chronic hepatitis .
d. HBV-X antigen represent the transformation to malignancy .
26.Which of the following drug over dose ,produce massive hepatic necrosis:
A. Aspirin B. Tetracycline
C. Retinol D. Paracetamol
27.A 55 year old male who is hepatitis C seromarker positive hase a firm, nodular
liver .All of the following findings can occur as a complications of this condition
except :
A. Hepatocellular carcinoma B. Coagulopathy
C. Ascites D. Hepatic infraction
28.The laboratory biopsy from 60 years old female with abdominal mass ,deep
jaundic and elevanted surum CEA reveals poorly differentiated glandular
structure which lined by atypical pleomorphic cells and surrounded by
desmoplastic stroma . The probable diagnosis for this finding is :
A. Gastric adenoma B. pancreatic carcinoma
C. Hepatocellular carcinoma D. Choristoma
29.A cute calculous cholecystitis occurs in the following condition except:
a. The postoperative state after major non biliary surgery
b. Sever trauma
c. Malnutrition
d. Sever burns
30.Liver biopsy from a 50 year old male reveal the presence of regenerative nodule,
dense fibrous septa with cholestasis proliferation of bile ducts which are most
likely to be:
a) Secondary biliary cirrhosis
b) Viral cirrhosis

‫ﻣﺮﻛﺰ اﻟﺨﻠﻴﺞ اﻟﻌﺮﺑﻲ‬ 23


Pathology Exams ‫ ﻃﺐ ﺑﺸﺮي‬32 ‫اﻟﻠﺠﻨﺔ اﻟﻌﻠﻤﻴﺔ ﺑﺪﻓﻌﺔ ﻧﺒﺾ اﻟﺤﻴﺎة‬

c) Alcoholic cirrhosis
d) Cardiac cirrhosis
31.A 55 year old male who is hepatitis C seromarkers positive has a firm, nodular
liver, all of the following findings can occur as complications of this condition
EXCEPT :
a) Hepatocellular carcinoma
b) Coagulopathy
c) Ascites
d) Hepatic infarction
32.The liver biopsy from a 50-year-old male with elevated serum alpha-fetoprotein
demonstrates loss of hepatic architecture with pleomorphism, hyperchromatic
nuclei , frequent mitotic figures. The probable diagnosis for these findings is:
a) Acute hepatitis A infection
b) Sclerosing cholangitis
c) Hepatocellular carcinoma
d) Hepatoblastoma
33.Regarding the seromarkes of HBV which the following is not correct :
A-Hbs ag refresnt glycol protein and detected in the serum
B-HBC AG represent core nucleocapid and remain in the infected hepatocyte
C-HBV-X protein represent precote and indicate ptogression chronic hepatitis
34.The following laboratory finding conjugated hyperbiliubunemia elevated serum
alkaline phosphatase increase bile acids and cholesterol are seen in:
A-Acute pancreatitis
B-liver schistomialsis
C-obstructive bilary diseases
35.The most common causes of jaundice include the following except :
A-Hemolytic anemias
B-Heapatitis
c-Autoimmune hepatitis
36.Regarding the primary malignancies of liver which of following is not carcinogenic :
A-vinyl cioride
B- HAV
C-Aflatoxin
37.The following are inborn errors of metabolism and pediatric liver disease except:
A-Hemochromatosis
B- Wilson disease
C-hepatoblastoma

‫ﻣﺮﻛﺰ اﻟﺨﻠﻴﺞ اﻟﻌﺮﺑﻲ‬ 24


Pathology Exams ‫ ﻃﺐ ﺑﺸﺮي‬32 ‫اﻟﻠﺠﻨﺔ اﻟﻌﻠﻤﻴﺔ ﺑﺪﻓﻌﺔ ﻧﺒﺾ اﻟﺤﻴﺎة‬

38.The following are the primary tumers of liver except:


A-mesothyloma
B-hepatoblastoma
C-Hepatoma
39.Elevated serum level of amylase and lipase are seen in:
A-Acute cholestasis
B-Acute heamorrhagic necrotizing pancreatitis
C-Cystic fibrosis of pancreas
40.Gall stone can be associated with the following conditions except:
A-ulcerative colitis
B-acute cholecystitis
C-secondary billary cirrhosis
41.The liver biopsy from 50 year old male with elevated serum alpha-fetoprotein
demonstrates loss of hepatic artiticture with pleomorphism hyperchromatic nuclie
the probable diagnosis for this findings is:
A-Acute hepatitis A infection
B-sclerosing choenergic
C-Hepatocelluler carcinoma
---------------------------------------------------------------------------------------------------------------------

Endocrine pathology:
Essays :
1. Write short account on goiter (Def. ,simple ,multinodular)
2. Hashimoto thyroiditis.
3. MEN syndrome.
4. Microscopic pictures of thyroid papillary carcinoma.
5. Complications of Hashimoto thyroiditis.
6. Classify malignant thyroid tumors.
7. Carcinoma of thyroid.
8. write short account on thyroid neoplasm.
9. Mention classical microscopic signs of papillary thyroid carcinoma?
10.Talk about thyroid carcinoma?
11.Hashimoto's thyroiditis?
12.Classify malignant of thyroid tumors
13.Carcinoma of thyroid
14.Enumerate the causes of pancytopenia
15.Enumerate the thyroid neoplasms benign and malignant .

‫ﻣﺮﻛﺰ اﻟﺨﻠﻴﺞ اﻟﻌﺮﺑﻲ‬ 25


Pathology Exams ‫ ﻃﺐ ﺑﺸﺮي‬32 ‫اﻟﻠﺠﻨﺔ اﻟﻌﻠﻤﻴﺔ ﺑﺪﻓﻌﺔ ﻧﺒﺾ اﻟﺤﻴﺎة‬

MCQs :

1. One of the following is characteristic feature of papillary carcinoma :


a) Many mitotic figures
b) Nuclear grooving
c) Prominent nucleoli
d) Hyperchromatic nuclei
2. Pheochromocytoma is :
a) Tumor of the cortex of the suprarenal gland.
b) Usually malignant tumor.
c) 10% bilateral.
d) Non of the above.
3. Follicular carcinoma of the thyroid gland :
a) The most common tumor of the thyroid gland.
b) Metastasizes early to the regional lymph nodes.
c) Affect mostly young aged men.
d) Non of the above.
4. Multi-nodular goiter is :
a) Benign enlargement of the thyroid gland.
b) Affects women more than men.
c) Can cause superior vena caval syndrome.
d) All of the above.
5. The following are true about non hodgkin's lymphoma:
a) Malignant proliferation of lymphoid tissue.
b) Associated with clinical manifestations like fever and preuritis.
c) Burkkit lymphoma is of high grade type.
d) D-T cell types have usually worse prognosis.
6. One of the following is characterstic feature of papillary carcinoma:
A- Many mitotic figures
B- Nuclear grooving #
C- Prominent nucleoli
D- Hyperchromatic nuclei
7. Pheochromocytoma is:
A- Tumor of the cortex suprarenal gland
B- Usually malignant tumor
C- 10% bilateral
D- None of above

‫ﻣﺮﻛﺰ اﻟﺨﻠﻴﺞ اﻟﻌﺮﺑﻲ‬ 26


Pathology Exams ‫ ﻃﺐ ﺑﺸﺮي‬32 ‫اﻟﻠﺠﻨﺔ اﻟﻌﻠﻤﻴﺔ ﺑﺪﻓﻌﺔ ﻧﺒﺾ اﻟﺤﻴﺎة‬

8. Follicular carcinoma of thyroid gland is:


A- The most common tumor of thyroid gland
B- Metastasizes early to regional lymph nodes
C- Affect mostly young aged men
D- None of above #
9. Multinodular goiter is :
A- Benign enlargement of the thyroid gland
B- Affects women more than men
C- Can cause superior vena caval syndrome
D- All of above #
10.Invasive ductal carcinoma is :
A- The most cancer of the breast#
B- Usually arise from acini
C- Clinically named medullary carcinoma
D- By its pathogenesis fibroadenoma is p…

11.MEN ( multiple endocrine neoplasia ) is:


A- Invasive ductal carcinoma + invasive lobular carcinoma
B- Medullary breast carcinoma + pheochromocytoma
C- Pheochromocytoma + follicular carcinoma of thyroid gland
D- None of above #
medullary thyroid carcinoma ‫اإلجابة الثالثه خطاء النه‬

12.Dark hemorrhagic and slightly enlarged adrenals where found at autopsy in a


teenager who died only hours after presentation to the emergency room with
fever and sever headache .this appearance is most consistent with :
A. Idiopathic Addison's diseases B. Metastatic carcinoma
C. Meningococcemia D. Widespread tuberculosis

13.Diffuse non –toxic (simple ) goiter used to be common (endemic ) in mountain


region because of :
A. Vitamin D deficiency B. Lack of dietary calcium
C. Lack of dietary iodine D. Genetic lack of thyroid binding globulin

‫ﻣﺮﻛﺰ اﻟﺨﻠﻴﺞ اﻟﻌﺮﺑﻲ‬ 27


Pathology Exams ‫ ﻃﺐ ﺑﺸﺮي‬32 ‫اﻟﻠﺠﻨﺔ اﻟﻌﻠﻤﻴﺔ ﺑﺪﻓﻌﺔ ﻧﺒﺾ اﻟﺤﻴﺎة‬

14.A 29 years- old primgravida has placenta Previa with extensive blood loss an
shock during delivery ,she is most likely to have which of the following
problems:
A. Cushing's syndrome B. Gravis 's disease
C. Galactorrhea D. Sheehan syndrome
--------------------------------------------------------------------------------------------------------------------

Male genital system:


SA:
1. Describe the pathological features of benign prostatic hyperplasia .
2. Compare between seminoma and embryonic carcinoma .
3. Discuss prostatic carcinoma.
4. Gross and microscopic pictures of seminoma.
5. Discuss benign prostatic hyperplasia.
6. Write short account about seminoma.
7. Classify tumors of testis and write short notes about seminoma.

MCQ:

1. Prostatic carcinoma:
a) Appear in the center of the gland
b) Start as ill define lesion in the gland outer portion
c) The mode of diagnosis is the early symptoms
d) Arise from gland stroma
2. Prostate nodular hypertension:
a) Appear late in life
b) Usually start in the periphery of the gland
c) Common etiology of carcinoma
d) Due to hormonal imbalance

‫ﻣﺮﻛﺰ اﻟﺨﻠﻴﺞ اﻟﻌﺮﺑﻲ‬ 28


Pathology Exams ‫ ﻃﺐ ﺑﺸﺮي‬32 ‫اﻟﻠﺠﻨﺔ اﻟﻌﻠﻤﻴﺔ ﺑﺪﻓﻌﺔ ﻧﺒﺾ اﻟﺤﻴﺎة‬

Musculoskeletal pathology:
Essays:
1. Write short account on
2. Dermoid tumor – Morphology of chondrosarcoma – Morphology of
osteosarcoma.
3. Write short account on pathology and pathogenesis of osteoarthritis.
4. Write short account about osteoarthritis?
5. Leiomyoma (definition - morphology)
6. Write short notes on the classification of bone neoplasms.
MCQs :
1. Keloid is :
a) benign skin tumor
b) benign muscle tumor
c) neoplastic fibrous tissue
d) over growth fibrous tissue
2. A 13 years boy with pain in his left thigh is found to have a neoplasm of the
femur that radiologically is diaphyseal in location and in biopsy shows
numerous small round blue cells . the probable diagnosis is :
A. Chondrosarcoma B. Metastatic Adenocarcinoma
C. Ewing's sarcoma D. Neuroblastoma

3. The most common causative bacteria for a cute osteomyelitis is:


A. E. coli B. Staphylococcus aureus
C. Proteus D. Sallmonella

‫ﻣﺮﻛﺰ اﻟﺨﻠﻴﺞ اﻟﻌﺮﺑﻲ‬ 29


Pathology Exams ‫ ﻃﺐ ﺑﺸﺮي‬32 ‫اﻟﻠﺠﻨﺔ اﻟﻌﻠﻤﻴﺔ ﺑﺪﻓﻌﺔ ﻧﺒﺾ اﻟﺤﻴﺎة‬

Central nervous system :


Essays:

1. Routes of intracranial infections.


2. Classification of primary intracranial tumors.
3. Laboratory finings of CSF in acute bacterial meningitis and acute lymphocytic
meningitis.
4. How the microbe reach the nervous system ?
5. Classification of CNS tumors.
6. Morphology of meningioma.
7. Morphology and chemical findings of viral meningitis.
8. Routs of intracranial infection
9. Classification of primary intracranial tumors
10.Write short account about brain abscess (causative organism ,morphology and
complication ).

MCQs :

1. The following statements are true about intracranial neoplasm EXCEPT :


a) Rarely have distance metastasis
b) Benign tumors behave biologically like malignant
c) Medulloblastoma is the disease of first decade of age
d) Secondary metastatic lesion usually unilocal
2. Which of the following is a neurological tumor : **
a) Schwannoma
b) Neurofibroma
c) Meningioma
d) Glioblastoma
3. Which of the following tumors had the best prognosis :
a) Medulloblastoma
b) Neuroblastoma
c) Pilocytic cytoma
d) Glioblastoma
4. The following are true about encephalitis :
a) May associated with meningitis
b) Focal necrotizing lesions produced by bacteria
c) Brain abscess is one of the examples

‫ﻣﺮﻛﺰ اﻟﺨﻠﻴﺞ اﻟﻌﺮﺑﻲ‬ 30


Pathology Exams ‫ ﻃﺐ ﺑﺸﺮي‬32 ‫اﻟﻠﺠﻨﺔ اﻟﻌﻠﻤﻴﺔ ﺑﺪﻓﻌﺔ ﻧﺒﺾ اﻟﺤﻴﺎة‬

d) All of the above


5. Acute bacterial meningitis characters by the following except.
a) meningeal irritation b) CSF show abundant lymphocyte
b) cloudy or purulent CSF d) increase protein level of CSF

6. The following futures about the neoplastic intracranial neoplasm except:


a) low cellularity b) giant cell
c) vascular endothelial proliferation d) nuclear and cytoplasmic polymorphism

7. All following statement about intracranial neoplasm are true except :


a) rarely have distance metastasis
b) benign tumor behave biologically like malignant
c) medulloblastoma is the disease of first decade of age
d) secondary metastasis lesions usually unilocal
8. The following are true about anaplastic intracranial neoplasm except:
a) low cellularity b)giant cells
c) vascular endothelial proliferation d)nuclear and cytoplasmic polymorphism
9. Flexner-wintersteiner rossett is microscopic feature of:
a) Retinoblastoma.
b) Neuroblastoma.
c) Immature sacrococcygeal teratoma.
10.Regarding neuroblastoma the following statements are true except:
a) 75% arise within abdomen.
b) Microscopical homer-wright psudorossett arrangement.
c) Elevated urine levels of vanillymandelle acid.
d) Associated with hypertension
11.Medulloblastoma:
a) Benign tumor of neuroglial cells
b) Primitive neuroectodermal #
c) Usually located in frontal lobe
d) May metastasize
12.A 60-years old female had a cerebral infraction. Months later a computed
tomographic (CT) scan shows a cystic area in her cerebral cortex .The CT
findings is a lesion that consequence of resolution from :
a) A. Liqufecative necrosis B. Apoptosis
b) C. caseous necrosis D. Atrophy

‫ﻣﺮﻛﺰ اﻟﺨﻠﻴﺞ اﻟﻌﺮﺑﻲ‬ 31


Pathology Exams ‫ ﻃﺐ ﺑﺸﺮي‬32 ‫اﻟﻠﺠﻨﺔ اﻟﻌﻠﻤﻴﺔ ﺑﺪﻓﻌﺔ ﻧﺒﺾ اﻟﺤﻴﺎة‬

13.Which of the following infectious agents is most likely the produce focal
necrotizing encephalitis :
a) A. Herpes simplex virus B. Toxoplasma gondii
b) C. Cytomegalovirus D. Bacteroides

14.Examination of CSF from lumber puncture shows 2 RBC's , 34000 WBC's ,


glucose of 20 mg/dl , and protein of 105 mg/dl .Which of the following
additional test would be the most helpful :
a) A. Cryptococcal antigen B. A cid fast stain
b) C. Gram stain D. India ink

15.A 65 years old male has been healthy all his life until a sudden seizure
.Neurologic exam reveals no focal abnormalities .A CT scan reveals poorly
demarcated large mass with central necrosis in the right frontal lobe .The most
likely diagnosis is :
a) A. Glioblastoma multiform B. Medulloblastoma
b) C. Low grade astrocytoma D. Meningioma

16.The most common case of dementia in the elderly population is :


a) Parkinson disease.
b) Alzheimer disease
c) Dysmyelinating disease.
d) Huntington disease

‫ﻣﺮﻛﺰ اﻟﺨﻠﻴﺞ اﻟﻌﺮﺑﻲ‬ 32


Pathology Exams ‫ ﻃﺐ ﺑﺸﺮي‬32 ‫اﻟﻠﺠﻨﺔ اﻟﻌﻠﻤﻴﺔ ﺑﺪﻓﻌﺔ ﻧﺒﺾ اﻟﺤﻴﺎة‬

Hematopoietic and Lymphoid:


Essays:
1. Write short account on the morphology of sickle cell anemia.
2. Enumerate the types of acute myeloblastic leukemia?
3. What are clinical features of acute leukemia?
4. What are the component of chronic myelopeoliferative disorder?
5. What are type of HD-rye classification?
6. Discuss pathophysiology of multiple myeloma?
7. Enumerate the types of acute Myeloblastic leukemias.
8. Enumerate the causes of iron deficiency anemia.

MCQs :

1. A 30-year-old male noticed a progressive cough for month. On physical


examination few small lymph nodes were palpable in the axillae, and the tip of
the spleen was palpable. A CBC showed , Hb 10.2 , Hct 31.1 MCV 90 , WBC count
67000 and platelet count 36000. If blasts with Auer rods are seen in the
peripheral blood smear , then the most likely diagnosis is:
a) Acute myeloblastic leukemia
b) Multiple myeloma
c) Chronic lymphocytic leukemia
d) Acute lymphoblastic leukemia
2. Regarding Hodgkin’s lymphoma , which of the following is true :
a) Neoplastic disorder of the histocyte
b) T-cell type has worse prognosis
c) Lymphocyte depletion is the most common type
d) Lymphocyte predominant type has excellent prognosis
3. Painless lymphadenopathies in 5 years old anemic child consistent with :
a) Burkitt’s lymphoma
b) Acute myeloblastic leukemia
c) Myelodisplastic disorder
d) Acute lymphoblastic leukemia
e) The appearance of a “Target cell” on examination of the peripheral blood
smear

‫ﻣﺮﻛﺰ اﻟﺨﻠﻴﺞ اﻟﻌﺮﺑﻲ‬ 33


Pathology Exams ‫ ﻃﺐ ﺑﺸﺮي‬32 ‫اﻟﻠﺠﻨﺔ اﻟﻌﻠﻤﻴﺔ ﺑﺪﻓﻌﺔ ﻧﺒﺾ اﻟﺤﻴﺎة‬

4. What is the type of lymphoma shows the following morphology :


(Enlarge gray white lymph node – diffuse lymphoblastic cells )
a) High grade lymphoma
b) Intermediate grade lymphoma
c) Low grade lymphoma
d) Burkitt’s lymphoma
5. Anemia ,bleeding disorders and infections are seen in the following diseases
except:
a) Aplastic anemia
b) Hypersplenism
c) Leukemias
d) Iron deficiency anemia
6. The following are myeloproliferative disorders except:
a) Chronic myeloid leukemia
b) Chronic lymphocytic leukemia
c) Polycythemia vera
d) Essential thrombocythemia
7. The following features lode of weight low grade fever classical RS cells and
inflammatory cells are seen in:
a) Lymphocytes depletion HD
b) Lymphocyte predominant HD
c) Nodular sclerosis HD
d) Mixed cellularity HD
8. Regarding seromarkers of HBV, which of the following statement is not correct :
a) Hbs Ag represent glycoprotein and detected in the serum
b) HBC Ag represent core nucleocapsid and remain in the infected hepatocyte
c) HBV-X protein represent precore and core region antigen and indicate
progression to chronic hepatitis
d) IgM anti-HBV indicate hepatocyte destruction after months followed by IgG
anti-HBV
9. Periodic acid shift (PAS) is positive in the following leukemias except :
a) M6
b) M2
c) ALL
d) L3-ALL

‫ﻣﺮﻛﺰ اﻟﺨﻠﻴﺞ اﻟﻌﺮﺑﻲ‬ 34


Pathology Exams ‫ ﻃﺐ ﺑﺸﺮي‬32 ‫اﻟﻠﺠﻨﺔ اﻟﻌﻠﻤﻴﺔ ﺑﺪﻓﻌﺔ ﻧﺒﺾ اﻟﺤﻴﺎة‬

10.Typhoid fever mostly characterized by :


a) Polycythemia
b) Leucopenia
c) Leukocytosis
d) Non-of the above
11.Sickle cell anemia mean :
a) Autosomal recessive hemolytic anemia effect the children in the first year
b) Replacement of guanine by valine
c) Hemolytic anemia caused mainly by enzyme mutation
d) Aggregation of unstable hemoglobin in erythroid cells

12.Lymphoepitelioma is characterized by all of the following except :


a) High incidence in young Asian
b) Rapid growth
c) Lymphoid and epithelia elements
d) High rate cure the surgery
13.Malignant thymoma is :
a) Neoplasm of lymphoid tissue
b) Neoplasm of epithelial tissue
c) Hodgkin’s disease of thymus
d) T-cell lymphoma

14.Lymphadenopathy is a feature of all of the following conditions EXCEPT :


a) Human immunodeficiency virus infection
b) Toxoplasmosis
c) Infectious mononucleosis
d) Non-Hodgkin’s lymphoma
e) Multiple myeloma
15.Virchow’s triad include except :
a) Endothelial injury
b) Hypercoagulability
c) Fibrinogen hyperactivivty
d) Turbulence of bold flow

16.The following can cause iron deficiency anemia except:


a) low dietary intake. b) acute blood loss. c) male absorption syndrome

‫ﻣﺮﻛﺰ اﻟﺨﻠﻴﺞ اﻟﻌﺮﺑﻲ‬ 35


Pathology Exams ‫ ﻃﺐ ﺑﺸﺮي‬32 ‫اﻟﻠﺠﻨﺔ اﻟﻌﻠﻤﻴﺔ ﺑﺪﻓﻌﺔ ﻧﺒﺾ اﻟﺤﻴﺎة‬

17.The following are symptoms of anemia except:


a) Pallor of skin
b) Increase blood rate
c) Normal breath
d) Palpitations
18.In thalassemia the following are true except.
a) deletion of gene b) absence production of gene
c) reduction in production of gene d) uncommon cause of anemia
19.Sickle cell anemia crisis are characterized by the following except:
A) painful b) bone narrow infection c) renal failure d) vascular occlusion
20.Anemia-bleeding and disorders are seen in the following disease except:
A)aplastic anemia B)hypersplenism C)leukemia D)iron deficiency anemia
21.The following are myelopeoliferative disorder except :
a) Chronic myeloid leukemia
b) Chronic lymphocytic leukemia
c) Polycythemia vera
d) Essential thrombocythemia
22.The following features -loss of weight -low grade fever-classical RS and
inflammatory cell are seen in:
a) lymphocytes depletion HD
b) lymphocytes predominance HD
c) nodular sclerosis HD
d) mix cellularity HD
23.The following can cause iron deficiency anemia except:
a) low dietary intake b) acute blood loss c)malabsorption syndrome
24.The following are symptoms of anemia except :
a)pallor of skin b)increase pulse rate c)normal breath d)palpitation
25.In thalassemia the following are true except :
a)deletion of gene b)reduction in production of gene
c)absent production of gene d)uncommon cause of anemia
26.Sickle cell anemia crisis are characterized by following except :
a)painful b)vascular occlusion c)bone marrow infraction d)renal failure
27.Malignant thymoma is:
a) Neoplasm of lymphoid tissue
b) Neoplasm of epithelial tissue#
c) Hodgkin’s disease of thymus
d) T cells lymphoma

‫ﻣﺮﻛﺰ اﻟﺨﻠﻴﺞ اﻟﻌﺮﺑﻲ‬ 36


Pathology Exams ‫ ﻃﺐ ﺑﺸﺮي‬32 ‫اﻟﻠﺠﻨﺔ اﻟﻌﻠﻤﻴﺔ ﺑﺪﻓﻌﺔ ﻧﺒﺾ اﻟﺤﻴﺎة‬

28.Lymphadenopathy is the feature of all the following conditions except :


a) Human immune deficiency virus infection
b) Toxoplasmosis
c) Infectious mononucleosis
d) Non-Hodgkin’s lymphoma
e) Multiple myeloma#
29.Sickle cell anemia mean:
a) Autosomal recessive hemolytic anemia effect the children in first year
b) Replacement of guanine by valine
c) Hemolytic anemia caused by enzyme mutation
d) Aggregation of unstable hemoglobin in erythroid cells
30.The following are true about non-Hodgkin’s lymphoma:
a) Malignant proliferation of lymphoid tissue
b) Associated with clinical manifestation like fever and pruritus
c) Burkkit lymphoma is of high grade type
d) D – T cell types have usually worse prognosis

31.The following can cause iron deficiency anemia except:


a)low dietary intake b)acute blood loss
c)male absorption syndrome

32.The following are symptoms of anemia except:


a)pallor of skin b)increase pulse rate
c)normal breath d)palpitation

33.In a thalassemia the following are true except:


a)deletion of gene b)reduction in production of gene
c)absent production of gene d)uncommon cause of anemia

34.Sickle cell anemia crisis are characterized by the following except:


a)painful b)bone marrow infarction
c)vascular occlusion d)renal failure

35.Anemia bleeding disorders and infection are seen in the following diseases
except:
a)aplastic anemia b) leukemia
c)hyperspleenism d) iron deficiency anemia

‫ﻣﺮﻛﺰ اﻟﺨﻠﻴﺞ اﻟﻌﺮﺑﻲ‬ 37


Pathology Exams ‫ ﻃﺐ ﺑﺸﺮي‬32 ‫اﻟﻠﺠﻨﺔ اﻟﻌﻠﻤﻴﺔ ﺑﺪﻓﻌﺔ ﻧﺒﺾ اﻟﺤﻴﺎة‬

36.A 30 year-old male noticed a progressive cough for one month .On physical
examination, a few small lymph nodes were palpable in the axilla , and the tip
of the spleen was palpable .A CBC showed :
a) Hb 10.2
b) Hct 31.1
c) MCV 90
d) WBC count 67000 and platelet count 36000 .
37.If blasts with Aure rods are seen in peripheral blood smear , then the most likely
diagnosis is :
A. Acute myeloblastic leukemia
B. multiple myeloma
C. Chronic lymphocytic leukemia
D. acute lymphoblastic leukemia

38.Regarding Hodgkin's lymphoma , which of the following is true :


a) Neoplastic disorder of histiocytes
b) T-cell type have worse prognosis
c) Lymphocyte depletion is the most common type
d) Lymphocyte predominance type have excellent prognosis

39.The appearance of a "Target cell" on examination of the peripheral blood smear


is most cosistent with organ from a patient who has :
a) Adenocarcinoma of the colon
b) A diagnosis of beta-thalassemia
c) Septecemia with E.coli
d) Hereditary spherocytosis
40.Painless lymphadenopathies in 5 years old anemic child consistent with :
A. Burkitt's lymphoma B. Acute myeloblastic leukemia
C. Myelodispalstic disorder D. Acute lymphoblastic leukemia

41.Anemia –bleeding disorder and infections are seen in the following diseases
except :
A. Aplastic anemia B. leukemias
C. Iron deficiency anemia D. Hypersplienism

42.A 35 year old anemic female presents with pneumonia ,and epistaxis with
several purpura . physical examination reveal no organomegaly . bone marrow

‫ﻣﺮﻛﺰ اﻟﺨﻠﻴﺞ اﻟﻌﺮﺑﻲ‬ 38


Pathology Exams ‫ ﻃﺐ ﺑﺸﺮي‬32 ‫اﻟﻠﺠﻨﺔ اﻟﻌﻠﻤﻴﺔ ﺑﺪﻓﻌﺔ ﻧﺒﺾ اﻟﺤﻴﺎة‬

aspiration demonstrates dry tap and bone marrow biopsy shows yellow marrow
with bands of fibrous tissue and sheets of lymphocytes . Which set of peripheral
blood estimation findings is most likely present :
a) CBC with Hb 8 gm /dl , WBC total 2000 per microliter, platelet count 3000 per
microliter .
b) CBC with Hb 20 gm/dl , Hct 61%, and MCV 92.
c) Total lymphocyte count of 2oo per microliter , WBC 12000 per microliter.
d) CBC with Hb 10 gm /dl , HCT 30%, MCV 85 ,platelet count 300000 per microliter.

43. The following are true regarding warm antibody immunohaemolytic anemia
except:
a)caused by IgG b)rarely caused by IgA c)caused by IgM d)activated at 37c

------------------------------------------------------------------------------------------------------------------
GIT pathology
Essays:

1. What are the complication of acute and chronic cholecystitis?


2. What are the risk factor for hepatocellular carcinoma?
3. Possible findings in an appendix?
4. Enumerate the causes and complication of portal hypertention?
5. Mention the clinical syndromes produce by viral hepatitis?
6. Discuss the morphology of viral cirrhosis?
7. Describe the pathogenesis of acute pancreatitis?
8. How can we distinguish the early gastric cancer?
9. Define and mention the clinical features "CHOIESTASIS" Colorectal Adenomas?
10.Polyps of the colon and their relation to malignancy?
11.predisposing factors and morphology of gastric cancer?
12.compare in tabulated form between benign and malignant ulcer.
13.write short account on the neoplastic colorectal polyps.
14.possible findings in an appendix enumerated.
15.compare in tabulated form between intestinal type and diffuse type of gastric
carcinoma.

MCQs GIT :

‫ﻣﺮﻛﺰ اﻟﺨﻠﻴﺞ اﻟﻌﺮﺑﻲ‬ 39


Pathology Exams ‫ ﻃﺐ ﺑﺸﺮي‬32 ‫اﻟﻠﺠﻨﺔ اﻟﻌﻠﻤﻴﺔ ﺑﺪﻓﻌﺔ ﻧﺒﺾ اﻟﺤﻴﺎة‬

1. Early gastric cancer :


A-confined to the mucosa only
B-can involve the musculariespropria
C-cannot be seen by the endoscopist
D-lymphe node may show metastasis
2. The more aggressive colonic polyp is:
a)villous polyp b)hyperplastic polyp
c)tubular polyp d)juvenile polyp

3. The following are causes of colorectal carcinoma except:


a)high dietary intake of fat
b)increase vegetable fibers
c)ulcerative colitis with dysplasis
d)familial polyposis

------------------------------------------------------------------------------------------------
Renal pathology :
ESSAYS:
1. Give an account on the microscopic picture of Grade1 of T.C.C of urinary bladder
(low malignant potential) ?
2. Discus the membranous glomerulonephritis ?
3. Describe the pathological picture(feature) of chronic pyelonephritis ?
4. Describe the pathological pictare of membranous glomerulonephritis ?
5. Renal transplant rejection ?
6. Enumerate primary glomerular disease, discuse one of them ?
7. Defined nephrotic syndrom and write an essay about lipoid glomerulonephritis
?
8. Causes of pylonephritis and write short note about the investigation ?
9. Causes of renal cell carcinoma ?
10.Classification of kidney tumer and write short notes about renal cell carcinoma
(morphology, causes,pathology, consequence) ?
11.Complication of Diabetes Melitis on kidney ?
12.Systemic Lupus erythmatuses in kidney ?
13.White short note about Pylonephritis ?
14.Write an essay about chronic Pylonephritis ?

‫ﻣﺮﻛﺰ اﻟﺨﻠﻴﺞ اﻟﻌﺮﺑﻲ‬ 40


Pathology Exams ‫ ﻃﺐ ﺑﺸﺮي‬32 ‫اﻟﻠﺠﻨﺔ اﻟﻌﻠﻤﻴﺔ ﺑﺪﻓﻌﺔ ﻧﺒﺾ اﻟﺤﻴﺎة‬

MCQs
1- The following are true about pathogenesis of primary glomerulonephritis except :
a) Circulating immune complex
b) Cytotoxic antibodies
c) Ag /Ab deposition
d) Circulatory bacteria
2-The following are true about acute glomerulonephritis except :
a) Clinically named acute nephritis
b) Glomeruli are focally involved
c) Leads to azotemia and polyuria
d) Biopsy is the diagnostic method
3-One of the following is not a feature of nephrotic
syndrome :
a) Mild proteinuria
b) Hypercholesterolemia
c) Hypoalbuminemia
d) Retention of sodium and water
4- Acute pyelonephritis :
a) Usually due to blood borne infection
b) Polymorphurine casts accasionally present
c) Non suppurative nephritis
d) Commonly due to bacteria
5- Renal calculi :
a) Main cause of renal failure
b) Can be due to hyperurecemia
c) All type are radio opaque
d) 25% are calcium containing stones
6- Morphological signs of glomerulonephritis is :
a) Tubular necrosis
b) Interstitial fibrosis
c) Increase glomerular cellularity
d) Rupture ascending
7- Morphology of pyelonephritis :
a) Interstitial inflammation
b) Increased glomerular mesangium
c) Cortical fibrosis
d) Hyperplasia

‫ﻣﺮﻛﺰ اﻟﺨﻠﻴﺞ اﻟﻌﺮﺑﻲ‬ 41


Pathology Exams ‫ ﻃﺐ ﺑﺸﺮي‬32 ‫اﻟﻠﺠﻨﺔ اﻟﻌﻠﻤﻴﺔ ﺑﺪﻓﻌﺔ ﻧﺒﺾ اﻟﺤﻴﺎة‬

8- Acute nephritic syndrome :


a) Due to aflatoxine
b) End- stage leidney
c) Causes polyurea
d) Leads to anurea
9- Chronic renal failure :
a) Developed in short time
b) Renal function remain normal
c) The patients have normal urine
d) Lead to end stage kidney
10- Renal calculi are :
a) Uric acid 70%
b) Calcium contain stone 75%
c) Cystine 50%
d) Mixed material (phosphate and magnesium) 95%
11- Renal cell carcinoma :
a) Invades renal veins
b) Metastasize to other kidney
c) Small round cell neoplasm
d) Never give metastasis
---------------------------------------------------------------------------------------------------------------------

Female genital system


Write short account on:
1. Cervical intraepithelial neoplasia (classification schemes and clinical significance
2. Fibro-cystic disease
3. Endometriosis (definition ,theories ,common sites and complications)
4. Endometrial hyperplasia (classification scheme , pathological diagnosis and it's
clinical significance)
5. Epidemiology of breast cancer
6. Follicular cyst of ovary
7. Causes of uterine leiomyoma
8. Fibroadenoma of the breast ( definition, morphology and significance
9. Non-neoplastic cysts of ovary (types, morphology and prognosis)
10.Leiomyoma (definition and morphology )
11.MEN syndrome
12.Microscopic picture of serous cyst adenoma of ovary

‫ﻣﺮﻛﺰ اﻟﺨﻠﻴﺞ اﻟﻌﺮﺑﻲ‬ 42


Pathology Exams ‫ ﻃﺐ ﺑﺸﺮي‬32 ‫اﻟﻠﺠﻨﺔ اﻟﻌﻠﻤﻴﺔ ﺑﺪﻓﻌﺔ ﻧﺒﺾ اﻟﺤﻴﺎة‬

13.Microscopic picture of fibro adenoma of the breast


14.An ovulatory cycle (causes, diagnosis and clinical significance)
15.Microscopic picture of endometrium polyp
16.Microscopic picture of simple endometrium hyperplasia

Defined the following :


(Adenomyosis- intracanalicular fibroadenoma – cin-1 - complex endometrium
hyperplasia)

Answer the questions:


1. Classify breast cancer
2. Enumerate the serous tumors of the ovary (types, morphology and prognosis)
3. Mention the prognostic factors in breast cancer with very brief comment on
each factor if applicable
MCQs in female G.S.
1-folllicular cyst
2-ovarian tumor>>> present late
3-endometrial hyperplasia >>> increase estrogen
4-liomyoma >>> encapsulated,, single or multiple
5-endometrial dating>>> 14day >>premenstrual cycle
6-CIN except >>> sexual transmitted disease
7-TZ all true except >>> go down in propriety
8-vaginal tumor >>> secondary more than primary
9-all are non-premalignant except >> herpes simplex
10-end cervical polyp

True and false


1. One of the benign ovary tumor is struma ovaril
2. The malignant surface epithelial ovary tumors show an infiltrative growth
3. Hemorrhage cyst is main pathological feature of ovary follicular cyst
4. Chorionic villi, atypical trophoblasts proliferation and myometrium invasion are
feature of choriocarcinoma
5. Partial vesicular mole associated with metastasis
6. Fallopian tube is very commonest site of ectopic pregnancy

‫ﻣﺮﻛﺰ اﻟﺨﻠﻴﺞ اﻟﻌﺮﺑﻲ‬ 43


‫أﺳﺌﻠﺔ اﻟﺪﻛﺘﻮر‬
‫أﻣﻴﻦ ﻋﻘﺒﺔ ﻓﻲ‬
‫& ‪CVS‬‬
‫‪Respiration‬‬
‫ﻣﺤﻠﻮﻟﺔ‬
l-,

Pathology Exams ‫ ﻃﺐ ﺑﺸﺮي‬32 ‫اﻟﻠﺠﻨﺔ اﻟﻌﻠﻤﻴﺔ ﺑﺪﻓﻌﺔ ﻧﺒﺾ اﻟﺤﻴﺎة‬

Questions and answer of cardiovascular systent

1- write short account and pathogenesis of :


A ) atherosclerotic Aneurysm:
* Abdominal Aortic Aneurysm (AAA) occurs in the descending part of the abdominal aorta
between the iliac and renal arteries. It is the most common Aneurysm. It is caused by
atherosclerosis
* Its pathogenesis:
Atheromatous internal nodules 7 compression of the underlying media and adventitia 7
atrophy and weakening of underlying layers 7 dilatation of the wall= atherosclerotic
Aneurysm

B) infective Endocarditis:
* It is an inflammatory disease of the endocardium resulting from hematogenous invasion of
the cardiac valves or mural endocardium by infective agents (mostly bacteria) spreading
from any infection in the body.
* Its pathogenesis:
(cardiac valves+ bacterial invasion) ·� ( bacterial deposition and proliferation on surface of
cardiac valves) 7 (destructive and necrotic change with damage to endothelial lining cells)
-,, (necrosis with platelets, fibrin and bacterial deposition) -,, ( thrombatic masses with
bacterial colonies= infective vegetation)-,, embolic complications -,, infarcts and distant
abscess. (Infective Endocarditis)

C) Atherosclerotic process
* it's a slow, progressive disease that may start in childhood. It can effect the arteries of the
brain, heart, kidney, the arms and legs. It's characterized by formation of focal atheiroma
(fibrofatty plaque) within intima leading to many complication events.
* It's pathological changes are:
a. early stage (non-progressive):
sub-internal lipid. deposition+ lipid-laden macrophage II Foamy Ce Us 11 -,, linear Fatty streak.
b. late stage {progressive):
Fatty steaks+ smooth muscle cell proliferation+ lipid-laden smooth muscle cells &
macrophages foamy cell"+ collagen & elastin deposition "7 Atheromatous nodule
II

"composed of "
�� �
superficial part= fibrous cap deeper part= necrotic core
= smooth m .cell, collagem, elastin = lipid-cholesterol clefts,
and foamy cells. foamy cells, calcium, fibrin &
necrotic depris.

c. complicated stage:
1- lschemia 2- aneurysm 3- pipestem arteries 4-ulcaration

‫ﻣﺮﻛﺰ اﻟﺨﻠﻴﺞ اﻟﻌﺮﺑﻲ‬ 44


h,I'-
Pathology Exams ‫ ﻃﺐ ﺑﺸﺮي‬32 ‫اﻟﻠﺠﻨﺔ اﻟﻌﻠﻤﻴﺔ ﺑﺪﻓﻌﺔ ﻧﺒﺾ اﻟﺤﻴﺎة‬

D) Myocardial infarction
* It's a clinical syndrome of IHD which i.s caused by prolonged myocardial ischemia and
characterized by ischemic myocardial necrosis & sever sudden prolonged chest pain
radiating to one or both arm, neck , jaw & back which is not by nitroglycerine'.
* lt 1s pathogenesis is sever coronary atherosclerosis and it is complication with sudden
thrombotic occlusion + coronary vasospasm � sever and acute myocardial ischemia '7
ischemic necrotic localized myocardial change '7 Ml.·

E) Angina pectoris:
* It's a clinical syndrome of IHD which is caused by transient myocardial ischemia �nd
.
characterized by severe substernal short and intermittent chest pain radiating to the left
arm, neck & jaw & upper abdomen relieved at rest or by using nitroglycerine.
* It's pathogenesis: is coronary atherosclerosis with gradual narrowing of the lumen 7
chronic myocardial ischemia without cell death and with atrophic and fibrotic changes '7
Angina.

F) Dissecting Aneurysm:
* It is Characterized by the formation of second lumen within the media of the B.V wall. It is
typically associated with hypertension and laminar medionecrosis. It is caused by genetic
factors.
* Its pathogenesis: degenerative and necrotic change in the media '7 loss of the elastic
fibers and smooth muscle cells with slightly destructed changes of the internal elastic lamina
7 laminar medionecrosis 7 weakening of the internal wall '7 internal tear formation by
the increased blood pressure 7 passing of blood from the lumen of the B.V in to the
necrotic media (through the internal tear) 7 blood dissects the necrotic media '7 cavity
filled with blood = second lumen.

2- Write short account on:


A) Rheumatic fever
Definition: It is an immune-mediated multi-systemic inflammatory disease of children
affecting the connective tissue of multiple organs including heart, skin, joints and CNS.
Cardiac manifestations:
1- Rheumatic pancarditis: involvement of all cardiac layers.
2- Rheumatic myocarditis : involvement of the interstitial myocardial fibrous tissue in
relation to the small B.Vs and between the heart muscle fibers .
* in severe cases � dilatation of cardiac chambers -? heart failure.
3- Rheumatic pericarditis : Sub-serosal fat+ aschaff bodies -? fibrin deposition on the
pericardia! layers '7 fibrinous pancarditis .
4- Rheumatic endocarditis: involvement of endocardium, especially the valves (valvular
closure) and the macCallum1s area on the posterior wall of the left atrium.
extra cardiac manifestations:
1) Joints involvement:
* Arthralgia = joint pain without clinical signs of inflammation.
* Arthritis = joint pain with clinical signs of inflammation.
* Polyarthritis = involvement of multiple Joints.

‫ﻣﺮﻛﺰ اﻟﺨﻠﻴﺞ اﻟﻌﺮﺑﻲ‬ 45


Pathology Exams ‫ ﻃﺐ ﺑﺸﺮي‬32 ‫اﻟﻠﺠﻨﺔ اﻟﻌﻠﻤﻴﺔ ﺑﺪﻓﻌﺔ ﻧﺒﺾ اﻟﺤﻴﺎة‬

2) skin involvement
* Subcutaneous nodules= small ,painless, swellings.
* Erythema marginatum= skin rash especially in extremities.
3) CNS involvement:
* Chorea= involvement muscular movements.

B)complications of MI
1- sudden death within 1-2 hours (20 % of patients).
2- cardiac arrhythmias within 2 weeks (80 % of patients).
3- Lt. vent. congestion failure "7 pulmonary edema (2 weeks).
4- cardiogenic shock( 15 % pa ents) with (2 weeks).
5- cardiac rupture (1-5 %).
6- thromboembolism (15-40 %) with (2 weeks)� distal infarctions.
7- fibrinous pancarditis (2-3 days).
8- cardiac aneurysm "7 rupture "7 death

C} Libman Sacks Endocarditis (LS E)


-It's a type of Endocarditis occurs in paUents with (SLE)
It characterized by formation of:-
Small sterile vegetation on either or both sides of the valve leaflets.
D) Arteriosclerosis
It means thickening and hardening of the blood vessel wall with loss of elastisity
lt1 s three types
a- Hypertensive arteriolosclerosis
b- Monckebergs sclerosis
c- atherosclerosis

E) Malignant hypertension
It's severe elevation of blood pressure and short fatal course
Usually affects young people and characterized by
*Morphological manifestations
In the kidney, he.art, eye, and brain a1nd the
Causes of death in malignant hypertiension are:-
Renal failure, cerebral infraction, myocardial infraction and heart failure

F) Stage of atherosclerosis
* It's pathological changes are:
a. early stage (non-progressive):
sub-internal lipid deposition+ lipid-laden macrophage II Foamy C�lls ". '"?' Linear Fatty streak.
b. late stage (progressive):
Fatty steaks+ smooth muscle cell proliferation+ lipid-laden smooth muscle cells &
macrophages 11 foamy cell 11 + collagen & elastin deposition '"?' Atheromatous nodule

‫ﻣﺮﻛﺰ اﻟﺨﻠﻴﺞ اﻟﻌﺮﺑﻲ‬ 46


Pathology Exams ‫ ﻃﺐ ﺑﺸﺮي‬32 ‫اﻟﻠﺠﻨﺔ اﻟﻌﻠﻤﻴﺔ ﺑﺪﻓﻌﺔ ﻧﺒﺾ اﻟﺤﻴﺎة‬

"composed of 11

� �
superficial part = fibrous cap deeper part = necrotic core
= smooth m .cell, collagen, elastin = lipid-cholesterol clefts,
and foamy cells. foamy cells, calcium, fibrin &
necrotic depris.

c. complicated stage:
1- lschemia 2- aneurysm 3- pipestem arteries 4-ulcaration

3-write short account on pathological picture of:


a- MI b- Rh. Fever
A- Ml: Gross picture: present of yellow, reddish necrotic area of soft constituency
surrounded by congested zone of granulation tissue.
Histological picture : three zones
1-infracted necrotic zone which is coagulative necrosis with lyses of nuclei and preservation
of outline.
2-congested zone proliferative capillary and fibroblast.
3- intact myocardium with degenerative change.
B- Rh. Fever: characterized by formation of characteristic lesions called Aschoffs bodies
which*microscopically consist of
1- foci of fibrinoid necrosis with collagen surrounded by inf. Cells
2-antischkow cells (cardiac histocyte) which reveal abundant basophilic cytoplasm and small
rounded central nuclei
3- Aschoff cells which an{ multinucleated giant
*macroscopically: the bodies are seldom seen by naked eye as oval graish nodules 1-3mm
in diameter .

4- what are the main difference between :·


a) Kawasaki and Takayasu's disease.

Affect small and medium size arteries ·:


especially coronary, skin , lymph Affect large, medium and
Site node, mucous membrane. small arteries including Aorta
(Mucocutaneous lymph node and its branches and arch.
s ndrome
Acute necrotizing and
Pathologic features Acute necrotizing inflammation
anulomatous inflammation
Thrombosis due to endothelial cell
Results
injury and aneurysm formation
Clinical sudden onset fever & mucocutaneous . - decrease or absence of
manifestations symptoms = (Erosion of oral mucosa pulse in both arms
and lips, skin rash, lymphadenitis & - ocular and neurologic
lymphadenopathy) features and hypertension
Most affected people Infants & children Young female

‫ﻣﺮﻛﺰ اﻟﺨﻠﻴﺞ اﻟﻌﺮﺑﻲ‬ 47


Pathology Exams ‫ ﻃﺐ ﺑﺸﺮي‬32 ‫اﻟﻠﺠﻨﺔ اﻟﻌﻠﻤﻴﺔ ﺑﺪﻓﻌﺔ ﻧﺒﺾ اﻟﺤﻴﺎة‬ ,''
;f,

b) Thromoangitis obliterans and polyarteritis nodosa.

Small and medium arteries adjacent to


small and medium arteries of
nerve and veins of the extremities
Site multiple organs (GIT, Kidney and
especially legs.
heart)
acute suppurative
Acute necrotizing and
inflammation.
Pathologic Chronic non-specific
Chronic non-specific
features inflammation
inflammation

formation of micro abcess and Formation of aneurismal nodules


thrombus. -,I.,
Results
Fibrosis with narrowing of Rupture with hemorrhage or
lumen � ischemia. thrombosis and ischemia

c) Transmural Ml and subendocardial Ml

Involve the whole thickness of left Involve � - Yi thickness of the left


ventricular wall by lschemic rnecrosis. ventricular wall by necrosis.
It's common.
Caused by atherosclerosis of one Uncommon.
coronary artey with critical stenosis. Caused by diffused and multifocal
atherosclerosis of all coronary
arteries without stenosis.

5- write short account on aneursm


( definition and it's pathological complications)
Def. :- is a disease that is characterized by a localized abnormal dilatation of blood vessels
wall or the wall of the heart that can follow Ml.
Complication may result in :-
a- Rapt�re of the blood vessels wall with fatal hemorrhage.
b- Endothelial cell injury with thrombosis ischemia change
c- Alteration in the blood flow.
d- compression of the surrounding tissue lead to atrophy, pain, erosion and occulasion
effect.

6- What do you understand by OR write by the def. of:­


a- Rh. vegetation in infective endoc;iirditis :
It's necrotic change with platelet and fibrin deposition characterized by multiple ,small and
non-friable thrombotic mass caused by inflamed valve and damage to endothelial covering
and it's the most characteristic feature for acute valvulitis.

‫ﻣﺮﻛﺰ اﻟﺨﻠﻴﺞ اﻟﻌﺮﺑﻲ‬ 48


Pathology Exams ‫ ﻃﺐ ﺑﺸﺮي‬32 ‫اﻟﻠﺠﻨﺔ اﻟﻌﻠﻤﻴﺔ ﺑﺪﻓﻌﺔ ﻧﺒﺾ اﻟﺤﻴﺎة‬

b- Marantic Endocarditis :
It's non bacterial thrombotic Endocarditis characterized by formation of small, sterile bland
thrombotic vegetation a long the line of closure of the valve leaflets .it affect previously a
normal valve.
* The etiology and path incompletely understood microscopically:-
The vegetation are with no inflammation cells or micro organism and only fibrin and
platelets are seen .and may cause peripheral embolization.
c- Transmural Ml :
It's ischemic necrosis in valves the full thickness of the ventricular wall from the
endocardium to epicardium and caused by thrombosis superadded to acute plaguevents.
- It's occurs in the distribution of single coronary artery.
- Most common .

7 -write short account on left side heart failure


(def. causes. And morphology)
-Def. It's HF that last from weeks to month and it's caused by slowly development factors
which affect left side of the heart
Causes:-
a- IHD especially Ml due to coronary .disease .
b- systemic hypertension with excessive work load on the heart.
c-Aortic valve disease.
d- Mitral incompetence.
e- Congenital heart disease.
f- Myocardial disease such as cardiomyopathy and myocarditis
* Morphology: manifested by hypertrophy and often dilatation of the left ventricle,
chronic venous congestion of lung, pulmonary edema and reduction in renal perfusion with.
1- chronic venous congestion of lung.
2- pleural effusion with hydrothorax often result.
3- reduction in renal perfusion.
4� cereral hypoxia.

‫ﻣﺮﻛﺰ اﻟﺨﻠﻴﺞ اﻟﻌﺮﺑﻲ‬ 49


Pathology Exams ‫ ﻃﺐ ﺑﺸﺮي‬32 ‫اﻟﻠﺠﻨﺔ اﻟﻌﻠﻤﻴﺔ ﺑﺪﻓﻌﺔ ﻧﺒﺾ اﻟﺤﻴﺎة‬

Questions and J'1nswer of Respiratory system


1)Write short account on extrinsic .Asthma
(definition ,cause and pathogenesis)'!
-It is the most common ,occur in children[mainly]and associate with a family history of allergy,
mediate by immune reaction;
-caused by environmental antigen including food dust.
-It is mediated by Type I hypersensitivity reaction involving lgE bound to FC receptors of
mast cell present on the surface of the lung epithelium->these lgE reacts with antigen -
>degranulation of mast cell->releasing of chemical mediators[Histamin]->:
1. increases vascular permeability.
2. bronchial muscle contraction.
3. s mula on of mucous glands->increases mucous secretion->narrowing of the lumen.

Z)Give the definition of the following:


Ghon complex ,Miliary T.B ,Emphyseina ,Centry ad.nar emphysema.

Ghon complex:
it's primary pulmonary complex characterized by combination of small Ghon foci at
periphery of the lung [Granulomatous inf.] with Hilar lymph node involvement as a result of
spread of infection from Ghon foci to draining LNs->causing:
-TB-!ymphadenitis
-TB-lymphangitis

Miliary T.B: it's the spread of T.B from the lung to multiple organs of the body due to blood
vessel invasion and it's most prominent in liver, bone marrow, spleen & kidney.

Emphysema: It is a dilation of the air spaces distal ·to terminal bronchioles with destruction
of their walls.

Centry acinar· emphysema :It is occurs in respiratory bronchioles {in the proximal/central
part of sinus},affect the upper lobe of lung,strongly associated with chronic bronchitis and
smoking.

3)Write short account on pathological picture of:


A-Bronchi.al Asthma/B-Lobar pneumonia

Brom:hial Asthma is characterized by:


1.Chronic in amma on of bronchial wall.
2.Hyperplasia of bronchial submucosal gland,
3.Hyperplasia of bronchial muscle.
4.Thickening of the basement membrane.
5.Mucous plugs in the air spaces forming thick sputum.

‫ﻣﺮﻛﺰ اﻟﺨﻠﻴﺞ اﻟﻌﺮﺑﻲ‬ 50


Pathology Exams ‫ ﻃﺐ ﺑﺸﺮي‬32 ‫اﻟﻠﺠﻨﺔ اﻟﻌﻠﻤﻴﺔ ﺑﺪﻓﻌﺔ ﻧﺒﺾ اﻟﺤﻴﺎة‬

Lobar pnemmmia is characterized by:


1- Present of intra-alveolar fibrenous & suppuritve exudates,
2- consolidation of large focal area or entire lobe.
Presence of four stages

stages Congestive Red hepatization Gray hepatization Resolution


occure Persists for 1-2 Persists for 2-4 Persists for 4-8
days days days
Histological Congested Alveoli filled with Fibrinosuppurative Digested
picture blood vessels fi brinohemorrhagic exudate exudates by
of alveoli exudate the action of
n.eutrop_hils
- - enzymes
grossly Heavy and Solid-darkened Solid-green brown Soft and pale
hyperaemic due to RBCs+fibrin due to decrease
lung RBCs and increase
neutrophils
enzymes

4)Write the n1ain differences between:


Extrinsic and Intrinsic Asthma

Extrinsic Asthm.a Intrinsic Asthma

Common uncommon

Caused by enviromnental antigens Caused by respiratmy tracts infection


includinn food amt dust

Occur in children and associate with Occur in adult,not associate withJ'amily


familv history of' a.llercrv history of allerav

Mediated by immune reaction Mechanism is not clear

Lobar and Branclwpneumcmia

Lobar pneumonia Bronchial pneumonia


Common uncommon
Affect adult especially male Affect all age,especially elderly and infants
Caused by streptococcus pneumoni and Caused by streptococcal and staphylococcal
klebsiella pneumoni bacteria
Characterized by the presence of intra- Characterized by the presence of intra-
alveolar fibrinous and suppurative e,<Udates alveolar suppurative only
Characterized by consolidation of large focal Characterized by multiple patches of
area or the entire lobe consolidation arollhd bronchioles
Microorganisms enter the lung and cause Microorganisms enter the lung and cause
inflammation of alveolar wall inflammation of bronchioles then extend to
alveoli
There are four stages There are no stages

‫ﻣﺮﻛﺰ اﻟﺨﻠﻴﺞ اﻟﻌﺮﺑﻲ‬ 51


Pathology Exams ‫ ﻃﺐ ﺑﺸﺮي‬32 ‫اﻟﻠﺠﻨﺔ اﻟﻌﻠﻤﻴﺔ ﺑﺪﻓﻌﺔ ﻧﺒﺾ اﻟﺤﻴﺎة‬

5)write short account on:


Lung adenocarcinoma ,SCCof the lung.

Jung adenocardnoma :not associated with s1noking ,common. in non smoker ,more
in female .characterized by peripheral location ,small size and slow growth .good
prognosis .associated with lung fibrous scar.
Histological picture :presence of glandular structure.

sec of the hmg [stJtmmous cell carcinoma]:


Associated with smoking and more common in male ,characterized by central location ,large
size ,slow growth and good prognosis.
Histological picture :presence of keratin parts and pleomorphic cells.

6)write on puln1onary embolism?


It is an import cause of sudden death.it occurs in patient with blood stasis11 venous stasis" .it is
caused by :Heart failure,severe trauma with fracture,prolonged bed rest,surgery in legs and
primary venous disease.
Most of pulmonary embolism arise from deep venous thrombosis in lower extremities and
pelvis.
Fragmented thrombi in lower extremities->emboli->to right side of the heart->pulmonary
embolism.
It is arise from diseases in the lower limb and there is three types:
1.small emboli;80%of cases,without clinical significance due to lysis.
· 2.moderate emboli:15o/c0f cases,10o/c0f moderate emboli causes pulmonary infarc on
especially with in patients with lung and heart diseases. 90%of moderate emboli causes no
infarction due to double blood supply of lung.
3.large emboli:5%of cases ,causes obstruction of the main pulmonary arterial trunk­
>sudden death.

7)write on characteristic features of primary PTB[Pulmonary


Tuberculosis]?
characterized by formation of 11 Ghon complex" which is :
1-small Ghon foci at the periphery of the lung.
2-hilar lymph node involvement as cl result of spread of infection from Ghon foci to draining
LNs->causing :TB-lymphadenitis ,TB··lymphangitis.

8)write on the progressive PTB?


It's uncommon, affected adult ,occur post primary and caused by
1- endogenous reac on of1ry lesion especially in pa ent with low immunity.
2- exogenous reaction by inhalation of bacilli.
It is characterized by:

‫ﻣﺮﻛﺰ اﻟﺨﻠﻴﺞ اﻟﻌﺮﺑﻲ‬ 52


Pathology Exams ‫ ﻃﺐ ﺑﺸﺮي‬32 ‫اﻟﻠﺠﻨﺔ اﻟﻌﻠﻤﻴﺔ ﺑﺪﻓﻌﺔ ﻧﺒﺾ اﻟﺤﻴﺎة‬

1-haemoptysis due to rupture of small blood vessels.


2-acute tuberculous bronchopneumonia due to involvement of bronchus and evacuation of
caseous material.
3-acute millary TB ,characterized by involvement of multiple organs of the body due to blood
vessels invasion.
4-TBulcers of intestine and larynx due to direct spread of infection by sputum.

9)write on pulmonary edema(definition,gross and histological


picture,mechanism)?
Def: accumulation of fluids in the alveolar spaces and interstitial tissue of the lung.
-Gross picture:lungs are bulky and heavy
Histological picture:
1-presence of slightly pink fluid in alvi:oli.
2-congestive capillaries.
3-presnce of heart failure cells hemosidrin-laden macrophages.
-Mechanism:
1- increase hydrostatic pressure due to left ventricular failure.
2- decrease osmotic pressure due hypoalbuminemia as a result ofnephrotic syndrome or
liver disease.
3- increase alveolar capillary permeability as a result of inf. Process of Jung.

1.0)Give the definiti.on of COPD and write short account on the


pathogenesis of emphysema?
-COPD (chronic obstructive pulmonary diseases) it's inf. Response of the lung to the
antigen lead to there obstruction and strongly associated with smoking and ,vantitrypsin
deficiency(inherited], includes :emphysema ,chronic bronchitis ,bronchial asthma and
bronchiectasis.

...pathogenesis of emphysema:

Inherited alpha antitrypsin deficiency smoking


/ �
deer. Anti-elastase activity oxidant in tobacco smoke increase macrophage
&neutrophil in alveoli

incr. elastase activity free redical release �


from neutrophil increase elastase
production

lung tissue damage

‫ﻣﺮﻛﺰ اﻟﺨﻠﻴﺞ اﻟﻌﺮﺑﻲ‬ 53


Pathology Exams ‫ ﻃﺐ ﺑﺸﺮي‬32 ‫اﻟﻠﺠﻨﺔ اﻟﻌﻠﻤﻴﺔ ﺑﺪﻓﻌﺔ ﻧﺒﺾ اﻟﺤﻴﺎة‬

11)what are the main differences between LCC and small CC?
LCC sec
Associated with smoking and common in Associated with smoking and more common
male in male
May has peripheral or central location It has central location
Characterized by bulky mass ,rapid growth Large size ,rapid growth,poor prognosis due
and poor prognosis due to early metastasis to early metastasis andextra-pulmonary
spread.
Very aggressive due to its bulky mass and Very. aggressive.
metastasis
Histological picture: Histological picture:
Presence of large ,pleomorphic anaplastic Presence of small ,dark-staining lymphocyte-
cells with multi-nucleated giant cells. like tumour
cells[small,pleomorphic,hyperchromatic]

‫ﻣﺮﻛﺰ اﻟﺨﻠﻴﺞ اﻟﻌﺮﺑﻲ‬ 54


Pathology Exams ‫ ﻃﺐ ﺑﺸﺮي‬32 ‫اﻟﻠﺠﻨﺔ اﻟﻌﻠﻤﻴﺔ ﺑﺪﻓﻌﺔ ﻧﺒﺾ اﻟﺤﻴﺎة‬

d)Myocardial Infarction.
It is caused by 2 main pathogenic factors :-
1-0cclusie thrombus overlying an ulcerated or fissured athermatous plaque
90% o cases..
2-Vsspasm,emboli, & unknown 10%.
e)Angina Pectoris.
Short myocardial ischemia without ischemic necrotic myocardial changes.
f)Destructing(Dissecting) Aneurysm..
-Formation of second lumen within the arterial wall
hypertension in the presence of cystic medial degeneration & necrosis
------... elastic tissue fragmentation & loss of smooth muscle
cells leading to mild destruction of the internal elastic lamina.
2-Write short account on :-
l)Rheumatic fever(definilion,cardiac &extra-cardiac manifestations).
-Deflnition:lmmune mediated multi-systemic disease,non-suppurative of
�hildren between 5-15 years.
-Cardiac manifestation:-
1-Pancarditis ·-----111a Involvement all 3 layers.
2-lnterstitial myocardium -------.. Fibrous tissue related to smo()th
blood vessels.
3-lnvolvement pericardium with fibrin deposition •
Fibrinous pericarditis ----+ Bread & butter appearance .
The fate of Bread & butter appearance is:-
1-Lysis

‫ﻣﺮﻛﺰ اﻟﺨﻠﻴﺞ اﻟﻌﺮﺑﻲ‬ 55


Pathology Exams ‫ ﻃﺐ ﺑﺸﺮي‬32 ‫اﻟﻠﺠﻨﺔ اﻟﻌﻠﻤﻴﺔ ﺑﺪﻓﻌﺔ ﻧﺒﺾ اﻟﺤﻴﺎة‬

Or 2- Fibrosis -+ Mediastinopericarditis.
Pericardia! milk spot
Adhesion pericarditis.

4- Involvement of endotarditls especially valves.


Acute -Formation of vegetation
valvulitis
- Mltral valve most affected

-Swelling & congested

Valve Fate

-Fibrotic
-Thickened-Deformed-Calcified due to
Feature\
Chronic valvular stenosis or incompetence�
valvulitis

-Extra-cardiac manifestation:-
1- Joint involvement: a-Arthralgia , joint pain b-Fleeting arthritis
c- Po.Jyarthritis

2-Skin involvement:
-Subcutanous nodules
-Erythema marginatum

3-CNS involvement :chorea -+ Involuntary muscular movement.

‫ﻣﺮﻛﺰ اﻟﺨﻠﻴﺞ اﻟﻌﺮﺑﻲ‬ 56


Pathology Exams ‫ ﻃﺐ ﺑﺸﺮي‬32 ‫اﻟﻠﺠﻨﺔ اﻟﻌﻠﻤﻴﺔ ﺑﺪﻓﻌﺔ ﻧﺒﺾ اﻟﺤﻴﺎة‬

2)Complications of Ml.
1- Sudden death within 1-2 hrs in 20% of patients.Often due to ventricular
fibriUation.
2-Cardiac arrhythmias in 80% Of patient within 2 weeks .
3-Left ventricular congestive failure ,pulmonary oedema 60% of patients
within 2 weeks.
4-Cardiogenic shock in 15% of patients within 2 weeks.
3)Ubman-sacks endocarditis.
-Type of endocarditis occurs in pateins with SLE & is characterized by
formation of small,sterile vegetations on either or both sides of the valve
leaflets.
4)Arteriosclerosis.
Def: Thickening & hardening of blood vessel wall.
Types:-
1-Arteriosclerosis.
2-Medial calcified sclerosis.
3-Atherosclerosis.
S)Malignant hypertension.
Def: severe elevation of blood pressure & short fatal course which affects
young people.

‫ﻣﺮﻛﺰ اﻟﺨﻠﻴﺞ اﻟﻌﺮﺑﻲ‬ 57


Pathology Exams ‫ ﻃﺐ ﺑﺸﺮي‬32 ‫اﻟﻠﺠﻨﺔ اﻟﻌﻠﻤﻴﺔ ﺑﺪﻓﻌﺔ ﻧﺒﺾ اﻟﺤﻴﺎة‬

Morphological manifestations :
a-In kidney ;malignant nephrosclerosis (hyperplastic changes&
necrotizing artriolitis),necrotic changes of glomerular cap.,rupture of
vessels &finally haemorrhage which appears as multiple foci on kidney
surface called(flea-bitten kidney).

b-Heart:mild left ventricular hypertrophy.


c-Eye:severe retinal haemorrhage & papillodema(odema of the optic disk).
d-Brain:cerebral haemorrhage.
&)Stages of atherosclerosis.
1-Early stage ..

Deposition of cholesterol & its esters in the subintimal surface C.T


----a.11i focal aggregation of foamy cells •
formation of fatty streaks.
2-Mlld stage.
Smooth muscle cells proliferate & synthesize collagen,elastin &
proteoglycans • deposition within the fatty streaks
--.........
• converted into mature fibrofatty atheroma called fibrous plaques
or fibrous nodules.
3-Complicated stage
Occurs when the pathological changes occur in fully developed
atheromatous plaques:-
1-lschemia due to reduce blood vessel lumen.

2-Pipestem arteries due to deposition Ca in necrotic area & thickened


intima.

3-Cholesterol emboli due to ulceration of fibrous gap.

‫ﻣﺮﻛﺰ اﻟﺨﻠﻴﺞ اﻟﻌﺮﺑﻲ‬ 58


Pathology Exams ‫ ﻃﺐ ﺑﺸﺮي‬32 ‫اﻟﻠﺠﻨﺔ اﻟﻌﻠﻤﻴﺔ ﺑﺪﻓﻌﺔ ﻧﺒﺾ اﻟﺤﻴﺎة‬

4-Superimposed thrombosis due to ulceration of fibrous cap which


followed by endothelial damage .
5-lntraplaque haemorrhage due to rupture of new capillaries.
6-Atherosderotic aneurismal.dilatation due to atrophy of underlying media

3.write short account on pathological picture of:


A-M.I:
Gross appearance:it appears as yellow-reddish irregular necrotic area of
soft consistency surrounded by congested hyperemic zone which is finally
replaced by granulation tissue..
Microscopic appearance:
a.{1-2)min �ATP levels fall, ce,ssation of contractility
b.(lOmin) 7 50% depletion of ATP, cellular edema, decreased membrane
potential and susceptibility to arrhythmias
c.(20-24)min'"71rreversible cell injury
B_Rheumatic fever:
Grossly look at different o_rgans cns,jolnt,heart valve
Microscopically: presence of specfic inflam. infiltrate called as.chaff
bodies which composed of:
1-foci of fibrinoid .necrosis with collagen surrounded by inflam. cells
2- antischkou's cells= cardiac histiocytes
3 ... aschoff multinucleated gaint cells

‫ﻣﺮﻛﺰ اﻟﺨﻠﻴﺞ اﻟﻌﺮﺑﻲ‬ 59


Pathology Exams ‫ ﻃﺐ ﺑﺸﺮي‬32 ‫اﻟﻠﺠﻨﺔ اﻟﻌﻠﻤﻴﺔ ﺑﺪﻓﻌﺔ ﻧﺒﺾ اﻟﺤﻴﺎة‬

4-What are the main differences between?


A)Kawasakl and Takayasu's disease

Takayasu disease Kawasaki d�ease Item


Occlusive thrombaortopathy, aortic arch Mucocutarteous lymph node Called
syndrome and pulse less disease. syndrome
Also called(iMantile polyarteritis
Iiodosa)
Young Asian women than males by 8: 1 Infants 4 small children The affected
people
Large and medium and small size arteries Small and medium sized Site
including :Aortic arch, whole aorta in 30%, muscular a(rteries especially
descending aorta in 10% of cases. corofiary artenes
Skin, L.Ns, imucus membrane
1-Acute stage: Granulomatous Acute necttizing vasculitis. Morphology
inflammation .with fibrosis & stenosis
2-Chronlc stage: Fibrosis & chronic
nonspecific inflammation.
1-Neurologic and ocular symptoms in form 1-acute onset of fever with skin Manifestation
of malaise. irash.
2-Decease or absent pulses in upper limbs. 2-conjuncti� and oral edema.
3-hypertension. 3-cervical l�mphoadenopathy.
4-with corop.ary involvement
lead to {Mlj, coronary artezy
aneurysm with rupture& sudden
death).
5-Is leading cause for acquired
heart disease in child.
Thrombosis and aneurysm Result
formation

‫ﻣﺮﻛﺰ اﻟﺨﻠﻴﺞ اﻟﻌﺮﺑﻲ‬ 60


Pathology Exams ‫ ﻃﺐ ﺑﺸﺮي‬32 ‫اﻟﻠﺠﻨﺔ اﻟﻌﻠﻤﻴﺔ ﺑﺪﻓﻌﺔ ﻧﺒﺾ اﻟﺤﻴﺎة‬ i.
·�
·ir.
,

B) Thromboangitis obliterans and Poly artritis nodosa

Polyarthritis nodosa Thromboangitis obliterans (Buerger's Items


- disease)
fn middle aged especially those Young male 20·30 years& associated .· Occurrence
with hepatits B infections. with using tobacco.
SmaJI and medium sized Small and medium-sized arteries of Sites
arteries. Rare in CNS. Common. extremities, mainly legs, veins &
in Kidney ,GtT& Heart. nerves.
According to the involved organ: 1-Exudative inflammation in form of Manifestations
Hematuria, GIT bleeding & microabcsesses.
ischemic heart disease. 2� by thrombus formation due to
endothelial cell damage and
hypercoagulability
3-fibrosis lead to narrowing
lumen-vascular insufficiency with
ischemic effect resulting in sever pain
& gangrene in the legs.
Has three stages: Acute suppurative and chronic Pathological
1-Acute stage: segmental non-specific inflammatory changes of feature
fibrinoid necrotizing B.V. ,veins, nerves.
inflammation.
2-Healing stage: Mononuclear
cell infilteration.
3-Healed stage: Nodularity by
fibrosis , fragmentation of
internal media& loss of internal
· elastic lamina.
Narrowing, micro aneurysm, Acute supp. lnfl. Formation of Result
rupture, hemorrhage, mi«:ro-abscesses and thrombosis
thrombosis and tissue ischemia Chronic non-specific infl.
Fibrosis with narrowing of lumen lead
to ischemia

‫ﻣﺮﻛﺰ اﻟﺨﻠﻴﺞ اﻟﻌﺮﺑﻲ‬ 61


Pathology Exams ‫ ﻃﺐ ﺑﺸﺮي‬32 ‫اﻟﻠﺠﻨﺔ اﻟﻌﻠﻤﻴﺔ ﺑﺪﻓﻌﺔ ﻧﺒﺾ اﻟﺤﻴﺎة‬

C) Transmural MI and Subendocardial infarction.


Subendocardial infarction(circumfer1mtial Transmural Ml
infarct)
1-0ccures in 10% of cases. 1-0ccures in 90% of cases.(most common)
2- Ischemic necrosis is limited to the inner 2-Ischemic necrosis involves the full
1/3 or 1/2 of ventricular wall. thickness
3-Caused by diffuse, stenosing Of the ventricular
atherosclerosis of all coronary arteries with wall(endocardium-epicardium)
no critical stenosis. 3-caused by thrombosis superadded to acute
plaque events.
4- occurs in distribution of a single coronary
artery

S.Wrlt� short account on aneurysm(def., pathological


complications)??

Def: aneurysm is a disease that is characterized by a localized abnormal


·,
dilatation of a blood vessel (artery or vien) or the wall of the heart (left
ventricular wall) that can follow a myocardial infarction. .Jt is caused by
disease leading to weakening of blood vessel wall
Complications:
1.Rupture of blood vessel wall with fatal hemorrhage.
2.Endothelial cell injury with thrombosis and ischemic changes.
3.Alteration in the blood flow.
4.Compression of the surrounding structures leading to the atrophy ,pain
,erosion and occlusion effects.

6.What do you understand by or Write the definition of:

‫ﻣﺮﻛﺰ اﻟﺨﻠﻴﺞ اﻟﻌﺮﺑﻲ‬ 62


Pathology Exams ‫ ﻃﺐ ﺑﺸﺮي‬32 ‫اﻟﻠﺠﻨﺔ اﻟﻌﻠﻤﻴﺔ ﺑﺪﻓﻌﺔ ﻧﺒﺾ اﻟﺤﻴﺎة‬

A.. Rheumatic vegetations In infective endocardltls:


friable, bulky single or multiple and may involve more than one valve
containing necrosis with fibrin, thrombus, inflammatory cells, and
microorganisms are present on the heart valves The aortic and mitral valves
are the most common sites of infection
B.Marantic endocarditis:
It is also called (non bacterial thrombotic endocarditis, is characterized by
formation of small, sterile, bland thrombatic vegetations along the line of
closure of the valve leaflets ..It affects previously normal valves especially
mitral valve and is associated with debilitating disorders. Such as metastatic
cancer and· other wasting conditions ,the exact etiology and pathogenesis
are incompletely understood.
C .Transmural M.I:
1.lschemic necrosis involves the full thickness of the ventricular wall from
the endocardium to the epicardium.
2. is caused by thrombosis superadded to acute plaque events.
3.lt occurs in the distribution of a single coronary artery.
4.most common 90% of cases..

7-Write short account on left side heart failure(def.,


causes, morphology).
Def.I Is CHF that lasts from weeks to months and is caused by slowly
development factors which effect the left side of the heart that's will lead to
unable to maintain the circulation for the needs of body.
Caused by:
1-IHDespecially MI due to coronary aa disease resulting in left ventricular
weakness.

‫ﻣﺮﻛﺰ اﻟﺨﻠﻴﺞ اﻟﻌﺮﺑﻲ‬ 63


Pathology Exams ‫ ﻃﺐ ﺑﺸﺮي‬32 ‫اﻟﻠﺠﻨﺔ اﻟﻌﻠﻤﻴﺔ ﺑﺪﻓﻌﺔ ﻧﺒﺾ اﻟﺤﻴﺎة‬

2-systemic hypertension with excessive work load on heart.

3-Aortic valve disease(stenosis or incompetence).

4-Mitral incompetence.

5-Congenital heart disease.

6-Myocardial disease as cardiomyopathy& myocarditis.

Morphology:
I-Hypertrophy and often dilation of the leftventricle.

2-Chronic venous congestion of the lung(brown induration)

3-pulmonary edema.

4-Reduction in renal perfusion.

5-Pleural effusion with hydrothorax.

6-Cerebral hypoxia.

‫ﻣﺮﻛﺰ اﻟﺨﻠﻴﺞ اﻟﻌﺮﺑﻲ‬ 64


Pathology Exams ‫ ﻃﺐ ﺑﺸﺮي‬32 ‫اﻟﻠﺠﻨﺔ اﻟﻌﻠﻤﻴﺔ ﺑﺪﻓﻌﺔ ﻧﺒﺾ اﻟﺤﻴﺎة‬

1-Write short account on extrinsic Asthma(definition, cause,


and pathogenesis ). ?

Def: it's occur in children mainly and associated with family history of
allergy, also it's most commcln , and is mediated by immune reaction.

Cause: by environmental antigen including food and dust.


Pathogenesis:
It's mediated by typel hypersensitivity reaction involving lgE bounded to
FC receptors of mast cell present on the lung epithelium�These lgE
reacts with antigen�Degranulation of mast cells�Releasing of chemical
mediators(mainly histamine} �1- increase in vascular permeability.
2-Broncial muscle contraction.
3-stimulation of mucous gland 7
increase mucous secretion -::), Narrowing of the lumen.

2;.Give the definition of the following:


. . .

1-Ghon complex: 11 lry pulmonary complex"


A-small Ghan foci at the periphery of the lung.(Granufomatous
inflammation)
B-Hilar lymph node involvement as a result of spread of infection Gihan
foci to drain LNs�causing : *T.B Jymphadenitis.
*lr.B.lymphangitis.

2-Miliary T.B: Characterized by involvement of multiple organs of the


body (due to Blood vessels invasion).
3-Emphysema: It's dilatation of the airs space distal to the terminal
bronchioles with destruction of their walls.
4-Centery acinar emphysema: it occurs in respiratory bronchioles ( in
the proximal part of sinus), affects the upper lobe of the lung, and
strongly associated with chronic bronchitis and smoking.

‫ﻣﺮﻛﺰ اﻟﺨﻠﻴﺞ اﻟﻌﺮﺑﻲ‬ 65


, I.

Pathology Exams ‫ ﻃﺐ ﺑﺸﺮي‬32 ‫اﻟﻠﺠﻨﺔ اﻟﻌﻠﻤﻴﺔ ﺑﺪﻓﻌﺔ ﻧﺒﺾ اﻟﺤﻴﺎة‬

3-Write short account on pathological picture of :

a-Bronchial asthma b- lobar pneumonia


a-Bronchial asthma is pathologically characterized by:
1-Chronic inflammation of bronchial wall.
2-Hyperatrophy of bronchial muscle.
3-Hyperplasia of the bronchial submucosal glands.
4-Thicking of basement membrane.
5-mucous plugs in the air spaces forming thick sputum.

b-Lobar pneumonia is pathologically characterized by :


1-lntra alveolar fibrinous and suppurative exudates
2-consolidation of large focal area or the entire lobe.
Name of Congestive Red hepatization Green or gray · Resolutoin
stage hepatization
Occurrence Persist.for Persist for 2-4 Persist for 4-8 days
1-2 days days
Histologically Congested Alveoli filled with Fi brinosuppurative Digested· exudates ·
blood fi rinohemorrhagic exudates by the action of
vessels of exudates neutrophils
alveoli enzyme

grossly Heavy and Solid-darkend= Solid- brown Due Soft and pale
hyperemic liver gross to decrease RBCS
lung appearance due and Increase
to RBC.s & fibrin neutrophils

‫ﻣﺮﻛﺰ اﻟﺨﻠﻴﺞ اﻟﻌﺮﺑﻲ‬ 66


Pathology Exams ‫ ﻃﺐ ﺑﺸﺮي‬32 ‫اﻟﻠﺠﻨﺔ اﻟﻌﻠﻤﻴﺔ ﺑﺪﻓﻌﺔ ﻧﺒﺾ اﻟﺤﻴﺎة‬

4-��Nrite the main differences between:


a-Extrinsic and Intrinsic asthma
Extrinsic asthma Intrinsic asthma
Common Uncommon
Caused by environmental antigens Caused by respiratory tract
including food and dust infections (especially viral one)
Occur in children mainly and Occur in adults , not associated with
associated with a family history of family history of allergy.
allergy
Mediated by immune reaction Mechanism is not clear.
b-Lobar and bronchopneumonia
-----------·----.-----------·
t---___ L_o_b_a_..;r p;..._�eumonia bronchopneumonia
Common Most common
Affects adult especially male Affects all ages especially elderly and infance
Caused by streptococcus pneumonia Caused by streptococcal and staphylococcal
and klebsiefla pn�umohia bacteria.
characterized by characterized by the presence of intra­
Intra alveolar fibrinous and alveolar suppurative exudates coming
suppurative exudates from bronchioles

characterized by characterized by multiple patches of


consolidation of large focal consolidation around bronchioles
area or the entire lobe

Microorganism enter the lung & Microorganism enter the lung and cause
cause inflammation of the alveolar inflammation of the bronchioles than extend to
wall alveoli.
They are 4 stages or phases No stages

5-Write short account on


Lung adenocarcinoma , sec of the lung
...Lung adenocarcinoma:not associated with smoking ,more in
female.it's characterized by p.eripheral location, small size and
slow growth.Good prognosis. Associated with lung fibrous scar
due to chronic inflammation.
*Histological picture: presence of glandular structure.

‫ﻣﺮﻛﺰ اﻟﺨﻠﻴﺞ اﻟﻌﺮﺑﻲ‬ 67


Pathology Exams ‫ ﻃﺐ ﺑﺸﺮي‬32 ‫اﻟﻠﺠﻨﺔ اﻟﻌﻠﻤﻴﺔ ﺑﺪﻓﻌﺔ ﻧﺒﺾ اﻟﺤﻴﺎة‬

-sec of the lung: Associate with smoking and more common in


male. It's characterized by cuntral location , large size and
growth and good prognosis.
-Histological picture: presence of keratin pearls+ pleomorphic
cells.
6-Write on pulmonary emboHsm?
-It is an important cause of sudden dt�ath.

-It occurs in patients with blood stasis.

-It is caused by:

1-Heart Failure 2-prolonged bed rest 3-Severe trauma with fracture

4-Surgery in legs S�Primary venous disease

-Most of pulmonary embolisms arise from(Deep Venous Thrombosis)(DVT) in the lower


extremities and pelvis..•Fragmented thrombi in lower extremities� Emboli�to right s.ide of
the heart�Pulmonary circulation�Pulmonarv embolism.

- Pulmonary embolism mostly arise from diseases in the lower limb and is of three types:

1-small emboli. 2-moderate embc•li 3-large emboli

7-Write on characteristic features of primary (PTB)


(Pulmonary Tuberculosis)???

Characterized by : formation of (primary pulmonary complex) ={Ghon


Complex)::
1-Small Ghan foci at the periphery of the I ung(granulomatous
inflammation)
2-Hilar lymph node involvement as a result of spread of infection from
Ghon foci to draining LNs, causing: 1-TB. Lymphangitis 2-TB.
Lymphadenitis.
It is usually asymptomatic due to healing by fibrosis and calcification of
primary TB lesions.

‫ﻣﺮﻛﺰ اﻟﺨﻠﻴﺞ اﻟﻌﺮﺑﻲ‬ 68


Pathology Exams ‫ ﻃﺐ ﺑﺸﺮي‬32 ‫اﻟﻠﺠﻨﺔ اﻟﻌﻠﻤﻴﺔ ﺑﺪﻓﻌﺔ ﻧﺒﺾ اﻟﺤﻴﺎة‬

8-Write on the progressive PTB???

The progressive pulmonary TB is characterized by :


1-Haemoptysis due to rupture of small blood vessels.
2-Acute Tuberculosis Bronchopneumonia due to involvement of
bronchus and evacuation of caseous material.
3-Acute Millary TB, characterized by involvement of multiple organs of the body (due to
blood vessels invasion)

4-TB ulcers of intestine and larynx due to direct spread of infection(by sputum).

9-Write on pulmonary edema (def, mechanism, gross and


histological picture)???

Def: accumulation of fluids in the alveolar spaces and interstitial tissue


of the lung.
It is caused by:

1-lncrease hydrostatic pressure due to left ventricular failure� due to


Ml or malignant hypertension.
2-Decrease osmotic (oncotic} pressure due to hypoalbuminemia�due to
nephritic syndrome or liver diseases.
3-lncrease alveolar capillary permeability as a result of inflammatory
process of the lung.
Pathological picture:

-Gross picture: Lungs are bulky and heavy


-Histolo1ical picture:

1-Presence of slightly pink fluid in alveoli


2-Congestive capillaries

3-Presence of (Heart Failure Cells)=hemosidrin-laden macrophages.

‫ﻣﺮﻛﺰ اﻟﺨﻠﻴﺞ اﻟﻌﺮﺑﻲ‬ 69


Pathology Exams ‫ ﻃﺐ ﺑﺸﺮي‬32 ‫اﻟﻠﺠﻨﺔ اﻟﻌﻠﻤﻴﺔ ﺑﺪﻓﻌﺔ ﻧﺒﺾ اﻟﺤﻴﺎة‬

10-Give the (def) of COPD and write short account on the


pathogenesis of Emphysema?

COPD: are diseases characterized by inflammation response leading to


lungs obstruction.these diseases are strongly associated with smoking
and alpha-antitrypsin deficiency(inherited) ...
COPD includes: 1- Emphysema 2-Chronic bronchitis 3-Bronchial
asthma 4-Bronchiectasis
-Pathogenesis of Emphysema:
Inherited alpha antitrypsin defidency Smoking

/ \ Increase macrophage &


Decrease anti elastase activity +--- oxidant in tobacco smoking

/l
neutrophils In alveoli


Increase elastase activity "- Free radicals release from
neutrophils
� Increase elastase production:

� Lung tissue damage

11-What are the main differences between LCC and


Small CC.?
LCC sec
Associated with smoking and Associated with smoking and more
common on male common in male
May has peripheral or central It has central location
location.
Characterized by bulky mass, rapid Characterized by rapid growt, large size
growth and poor prognosis due to poor prognosis due to early metastasis &
early metastasis extra pulmonary spread
Very aggressive due its bulky mass Very aggressive 4

and metastasis.
Histological picture: presence of Histological picture: presence of small dark-
large, pleomorphic anaplastic cells staining lymphocyte like tumor cells(small,
with multinucleated giant cells. pleomorphic, hyperchromatic.)

‫ﻣﺮﻛﺰ اﻟﺨﻠﻴﺞ اﻟﻌﺮﺑﻲ‬ 70

You might also like